MSK PA EASY

Lakukan tugas rumah & ujian kamu dengan baik sekarang menggunakan Quizwiz!

Shoulder dislocation commonly involves injury to which nerve? A Axillary B Median C Peroneal D Radial E Ulnar

Explanation The Correct Answer is: A The axillary nerve is in close proximity to the glenohumeral joint, thus making it vulnerable to injury during a shoulder dislocation. The median, radial, ulnar nerves are more distal in the upper extremity and and the peroneal nerve is in the leg. ( Roberts, Merck Manual Online Medical Library, 2007)

What is the most common joint dislocation in children? A Ankle B Elbow C Finger D Knee E Shoulder

Explanation The Correct Answer is: B In children the elbow is the most commonly dislocated joint and it is the third most common joint dislocation for adults. The shoulder and finger are dislocated more frequently than the elbow in adults. Knee dislocations at the femur-tibia joint are rare (patellofemoral dislocations are more common) and ankle dislocations are also relatively rare.

A 65-year-old man presents with complaints of acute onset of pain and swelling of the right great toe. He denies recent alcohol ingestion or trauma to the area. On physical examination, the patient is afebrile, and the first metatarsophalangeal joint is erythematous, swollen, and warm to the touch. Laboratory evaluation reveals a WBC (white blood cells) count of 12,000/μL and a normal differential. Serum uric acid level is found to be 5 mg/dL. Synovial fluid analysis reveals the presence of rhomboid-shaped crystals. Which of the following is the most likely diagnosis? A acute gout B pseudogout C psoriatic arthritis D infectious arthritis E rheumatoid arthritis

Explanation The Correct Answer is: B Pseudogout presents similarly to acute gout and is best diagnosed by the finding of the rhomboid-shaped crystals of calcium pyrophosphate in joint aspirates. Joints commonly involved in pseudogout are the knees and wrists and other joints such as the metacarpophalangeals, hips, shoulders, ankles, and elbows. The diagnosis of pseudogout is further supported by the finding of a normal serum uric acid level. Acute gout would more likely be associated with an elevated serum uric acid level. Psoriatic arthritis commonly presents with asymmetrical oligoarticular involvement of two to four joints, and in a higher percentage of patients, there is known presence of the dermatological expression of psoriasis. Infectious arthritis is ruled out with the findings of an afebrile patient and WBC count of 12,000/μL. In acute infectious arthritis, the WBCs would be expected to be elevated in the range of 50,000 to 200,000/μL. Rheumatoid arthritis usually presents with symmetrical polyarticular involvement of three or more joints. (Hellmann and Imboden, 2008, p. 709) Hellmann DB , Imboden JB. Arthritis and musculoskeletal disorders. In: Tierney LM , McPhee SJ , Papadakis MA, eds. 2008 Current Medical Diagnosis and Treatment. New York, NY: McGraw-Hill; 2008:703-756.

Which type(s) of Salter-Harris fractures can generally be treated with closed reduction and cast immobilization? A Type I B Types I and II C Types I, II, and III D Types I, II, III, and IV E Types I, II, III, IV, and V

Explanation The Correct Answer is: C Minimally displaced Salter-Harris types I, II, and III fractures generally can be treated with immobilization only. Types IV and V involve the cartilage of both the articular surface and the growth plate. To ensure proper alignment and a congruous joint surfaces open reduction and internal fixation is usually necessary. (Sarwark, Ed., Essentials of Musculoskeletal Care, 4 th Edition, 2010)

Which rotator cuff muscle is most commonly injured? A Infraspinatus B Subscapularis C Supraspinatus D Teres Major E Teres Minor

Explanation The Correct Answer is: C The supraspinatus is involved in abduction and external rotation of the shoulder joint. It is often injured in repetitive overhead activities such as swimming and throwing sports. While all muscles of the rotator cuff can be injured, the supraspinatus has been shown to sustain the most frequent injuries. The infraspinatus and teres minor are also involved in abduction and external rotation, while the subscapularis and teres major assist in internal rotation. The teres major is not considered a rotator cuff muscle. (Simons, Rotator Cuff Tendonopathy, UpToDate, 2010)

What types of connective tissue are injured in a strain? A Bones and muscles B Fascia and joint capsules C Ligaments and joint capsules D Muscles and tendons E Tendons and bones

Explanation The Correct Answer is: D A strain involves injury to the muscles and tendons that are responsible for active movement of various body parts. Fascia is a part of the muscle-tendon unit, so injury to fascia would be considered a strain as well. Injury to ligaments and joint capsules would be considered a sprain and damage to bone would be classified as a fracture. (Sarwark, Ed., Essentials of Musculoskeletal Care, 4 th Edition, 2010)

Which physical complaint is the leading cause of lost work time and disability for patients under the age of 45? A Ankle pain B Hip pain C Knee pain D Low back pain E Shoulder pain

Explanation The Correct Answer is: D Low back pain is the most common causes of lost time from work and disability. Factors that can lead to back problems include repeated or heavy lifting and twisting or use of vibrating equipment along with poor fitness levels, smoking, poor job satisfaction and other psychosocial factors. Fortunately, 80% of patients experience significant recovery within one month. All of the other types of pain can occur based on multiple factors, but none of them occur with a frequency that rivals low back pain. (Sarwark, Ed., Essentials of Musculoskeletal Care, 4 th Edition, 2010)

A 64-year-old female who has a history of injectable drug use presents with blood work that reveals leukocytosis with a left shift, and there is suspicion of osteomyelitis based on the patient's prior history. Based on this history, what bone would be most affected by hematogenous osteomyelitis in adults? A Feet B Long bones C Pelvis D Sternoclavicular bones E Vertebrae

Explanation The Correct Answer is: E Hematogenous osteomyelitis accounts for about 20% of all cases of osteomyelitis in adults. It is more common in males and the prevalence is higher amongst those who are IV drug abusers, patients being treated with dialysis or who have sickle cell disease. Other conditions which may lead to sepsis (i.e. patients with central lines, urinary infections, and urethral catheterization) increase the risk of hematogenous osteomyelitis. Unlike children, the long bones are rarely affected in adults with the vertebrae being the most likely location for the bone infection to occur. Lumbar vertebrae are most often affected, followed by thoracic and cervical vertebrae. Osteomyelitis of the sternoclavicular bones and pelvic bones are not uncommon sites, but these tend to be most frequent amongst IV drug abusers.

A 46-year-old male is evaluated for a fall from approximately 15 feet, landing on his legs. There is intense pain and swelling to the lower extremity, with decreased sensation. What bone is fractured and is most commonly associated with acute compartment syndrome? A Femur B Patella C Navicular D Talus E Tibia

Explanation The Correct Answer is: E The tibia is the most common bone fracture that leads to acute compartment syndrome. It can also occur after other long bone fractures of the arms and legs. While it does occur in the hands and feet, it is unlikely that the fracture of a small carpal bone (scaphoid) or tarsal bone (talus) would result in compartment syndrome. Damage to the hyoid bone in the neck and the patella in the knee are not associated with compartment syndrome.

The most commonly fractured long bone in both adults and children is which of the following? A Femur B Fibula C Humerus D Radius E Tibia

Explanation The Correct Answer is: E The tibia is the most commonly fractured long bone in the body for both adults and children. The fractures are often the result of sporting activities in the young and may occur from a simple fall in the elderly - especially those with osteoporosis. Motor vehicle accidents are another common cause of tibial fractures. Open or complex tibial fractures are sometimes associated with compartment syndromes, infection and neurovascular compromise. The femur is the strongest of the long bones and generally only sustains fractures when exposed to extreme stress, such as that experienced in a motor vehicle collision or industrial accident. Fibular fractures commonly occur with a direct below to the lateral lower leg or with extreme ankle rotational forces or excessive inversion. A high percentage of ankle fractures involve the fibula, especially in older women. Humerus fractures are relatively rare in adults, but are the second most common fractures to occur at birth - behind only the clavicle in frequency. The radius is the most commonly fractured bone in the upper extremity, but still less common in frequency than the tibia. Falls on an outstretched arm are a common mechanism for the injury. (Grazel, Editor, UpToDate, 2010)

Active elbow extension is primarily controlled by which muscle(s)? A Anconeus B Biceps C Brachialis D Brachioradialis E Triceps

Explanation The Correct Answer is: E The triceps are the primary muscles that produce active elbow extension. The anconeus muscle is known as a secondary elbow extensor. The biceps and brachialis muscles are primary muscles of elbow flexion and the brachioradialis is a secondary elbow flexor. (

In a boxer's fracture, the presence of how much angulation of the fifth metacarpal neck would require referral for reduction as opposed to treatment with a simple ulnar gutter splint? A Less than 10 degrees B 10-19 degrees C 20-29 degrees D 30-39 degrees E Greater than 40 degrees

Explanation The Correct Answer is: E When there is greater than 40 degrees of angulation or an extensor lag (the patient cannot fully extend the affected finger) then a referral is required. Lesser degrees of angulation without an extensor lag can generally be handled conservatively with an ulnar gutter.

A 66-year-old male patient complains of pain and swelling in his left foot intermittently over the past year. He denies any current symptoms. On examination you note the following findings. His left foot is unremarkable. Laboratory finding include an elevated uric acid. What is the most likely diagnosis? A rheumatoid arthritis B gout C osteoarthritis D septic arthritis E systemic lupus

Explanation The correct answer is (B). The patient has gouty tophi. His uric acid level is elevated, which further supports a diagnosis of gout. It is suspected that the intermittent left-foot swelling is related to acute flares of gout, which usually affects the first MCP joint. Choices (A), (C), (D), and (E) are not associated with elevated uric acid level or tophi. The patient may have osteoarthritis, but this does not explain the high uric acid level and the tophi on exam.

A 76-year-old male presents to the hospital with a hot, swollen, and painful right knee that appears to have an effusion. After your examination, you decide to perform an arthrocentesis. The fluid aspirate is turbid, cloudy, and most definitely looks like it is infectious material. Based on this clinical scenario, what would be most likely bacterial organism present in an elderly septic arthritis? A Hemophilus influenzae B Neisseria gonorrhoeae C Pseudomonas aeruginosa D Staphylococcus aureus E Streptococcus pneumonia

xplanation The Correct Answer is: D Septic arthritis is an infection within a joint space. The infection can be caused by multiple pathogens including bacteria, viruses, fungi and mycobacteria, but in older adults the most common organism implicated is Staphylococcus aureus. These bacteria can get into the joint space through multiple mechanisms including direct inoculation after a penetrating injury, hematogenous spread from an infection elsewhere in the body, or from extension from a contiguous bone infection. All of the bacteria mentioned as possible answers are possible causes of septic arthritis, but there prevalence varies within different patient populations. Hemophilus influenza and Streptococcus pneumonia are commonly seen in children less than five years of age that develop septic arthritis. Neisseria gonorrhoeae is most common in sexually active adolescents and young adults. Pseudomonas aeruginosa is a common cause of septic arthritis in IV drug abusers

A 65-year-old woman presents with severe mid-back pain of 2 weeks duration. She has no history of trauma. Radiographic evaluation reveals compression fractures of T11 and T12. A complete blood count, erythrocyte sedimentation rate, serum protein, serum calcium, phosphate, and parathyroid hormone levels are all within normal ranges. In addition to ordering a dual-energy x-ray absorptiometry (DEXA) scan, which of the following laboratory evaluations is most helpful in evaluating this patient for secondary causes of this presentation? A bone biopsy B rheumatoid factor C serum magnesium D 25-hydroxyvitamin D E antinuclear antibodies test

A 65-year-old woman presents with severe mid-back pain of 2 weeks duration. She has no history of trauma. Radiographic evaluation reveals compression fractures of T11 and T12. A complete blood count, erythrocyte sedimentation rate, serum protein, serum calcium, phosphate, and parathyroid hormone levels are all within normal ranges. In addition to ordering a dual-energy x-ray absorptiometry (DEXA) scan, which of the following laboratory evaluations is most helpful in evaluating this patient for secondary causes of this presentation? A bone biopsy B rheumatoid factor C serum magnesium D 25-hydroxyvitamin D E antinuclear antibodies test

A 78-year-old Caucasian female has a 3-year history of stiffness and achiness of bilateral shoulders and hips. She has been tested for rheumatoid arthritis in the past and has been found negative. Multiple radiographs of her hips and shoulders are unremarkable. She admits that she was placed on prednisone for an allergic reaction and noted a temporary resolution of her symptoms. For the past two weeks she complains of increasing symptoms now involving her neck and pain in her jaw with chewing. Today she noticed that her scalp is sore when she brushed her hair on the right side. What test is the gold standard for diagnosis of this patient's current symptoms? A ESR 60 mm/h B temporal artery biopsy C Elevated CRP D ultrasound of temporal artery E polymyalgia rheumatica (PMR) of temporal artery

Explanation B The correct answer is (B). This patient has long standing symptoms of polymyalgia rheumatica (PMR) with current symptoms suggestive of giant cell (temporal) arteritis. Temporal artery biopsy is considered the gold standard for diagnosis of giant cell (temporal) arteritis. Patients with temporal arteritis may have an elevated erythrocyte sedimentation rate (ESR) or CRP, but this is not required for diagnosis. A color ultrasound of the temporal artery will sometimes show edema or stenosis of the affected artery but is not very sensitive for giant cell arteritis. MRA is used for diagnosis of larger arteries with vasculitis and not routinely used in the diagnosis of temporal arteritis. (Imboden et al., 2007, Chapter 31)

An 84-year-old female is taking prednisone for the past year, secondary to a diagnosis of biopsy-confirmed giant cell arteritis. Her dose was tapered to 20 mg/day, but her symptoms returned several months ago and her dose was increased. In addition she is taking hydrochlorothiazide (HCTZ) for hypertension, but denies any other medications or medical problems. What treatment should you recommend to this patient at this time? A glucophage B simvastatin C calcium and vitamin D D fosamax E naprosyn

Explanation C The correct answer is (C). This patient requires chronic prednisone treatment. Common complications of chronic glucocorticoid treatment include osteoporosis and diabetes. At this point with this patient there is no information stating that she has diabetes or osteoporosis. Before initiation of treatment with glucophage, the patient should have a diagnosis of diabetes mellitus. Before initiation of fosamax the patient should have a bone density scan to look for osteopenia/osteoporosis. In the meantime, calcium supplementation with vitamin D is warranted for prevention of osteoporosis. Simvastatin is not indicated, and naprosyn may increase risk of gastrointestinal bleeding with chronic prednisone.

In a 4-year-old female child who presents with "toeing in," which of the following is the likely etiology? A femoral anteversion B genu valgum C genu varum D tibial torsion

Explanation The Correct Answer is: A "Toeing in" in children before the age of 2 is typically due to tibial torsion; however, any "toeing in" after the age of 2 to 3, is usually due to femoral anteversion. The femur has more internal rotation that results in the presentation. Genu varum is known as bowleg and genu valgum is known as knock-kneed.

A 20-year-old male presents to the emergency department complaining of pain to the right shoulder region while playing basketball. He states that his arm was pulled back and rotated while he was moving forward, and then felt a popping sensation in the shoulder. Since then he has not been able to move the shoulder at all due to pain and immobility. It is suspected that he has a dislocation. Given this scenario, what would be the most likely type? What type of shoulder dislocation did he most likely experience? A Anterior B Inferior C Multidirectional D Posterior E Superior

Explanation The Correct Answer is: A All of the various types of dislocations mentioned above are possible, but anterior dislocations are by far the most common (>95%) and they are the most common of all joint dislocations. Most occur as a result of a fall or other traumatic event and they may become recurrent. The shoulder is most susceptible to an anterior dislocation when it is abducted and externally rotated. The shoulder joint is considered a very mobile joint, but this also renders it very susceptible to injury. (Roberts, Merck Manual Online Medical Library, 2007)

Out of all cervical vertebrae, which two are responsible for the greatest amount of rotation? A C1 & C2 B C2 & C3 C C3 & C4 D C4 & C5 E C5 & C6

Explanation The Correct Answer is: A Approximately 50% of cervical rotation takes place between the C1 (atlas) and C2 (axis) vertebrae. These first two cervical vertebrae have a different shape from the other cervical vertebrae that allow for this greater range of motion. The remaining 50 % of cervical rotation is split fairly evenly between the remaining vertebrae. Approximately 50 % of flexion and extension occurs between the occiput at the base of the skull and C1 with the remaining 50% distributed fairly evenly between the remaining vertebrae with a slightly higher percentage occurring at the C5 & C6 level. (Hoppenfeld, Physical Examination of the Spine and Extremities, 1976)

Which of the following joints has the lowest occurrence rate of osteoarthritis? A Elbows B Hands C Hips D Knees E Spine

Explanation The Correct Answer is: A Because the elbow is not a weight bearing bone, the rate of osteoarthritis in the elbow is considerably less that what is found in locations like the hips, knees and spine. The hands have one of the highest rates of occurrence of osteoarthritis, likely due to their near constant use and propensity for minor (or major) injury. When elbow arthritis does develop it is often post-traumatic osteoarthritis related to a significant injury in the past that disrupted joint surface integrity or as a result of rheumatoid arthritis, a systemic illness. Osteoarthritis of the elbow will generally present with pain, stiffness, and decreased range of motion. Osteophytes that form on the medial elbow might be implicated should neurological symptoms develop that correlate with ulnar nerve distribution as this nerve does pass in close proximity to the elbow on the medial side. (Sarwark, Ed., Essentials of Musculoskeletal Care, 4 th Edition, 2010)

A 64-year-old female presents for an annual examination. She is 5'0" and weighs 92 pounds; compared to her examination 3 years ago she has lost an inch in height. After performing a dual-energy x-ray absorptiometry (DXA) scan the diagnosis of osteoporosis is confirmed. Which of the following supplements should be recommended? A Calcium carbonate 1,000 mg and vitamin D 800 international units daily B Calcium carbonate 500 mg and vitamin D 2,500 international units daily C Calcium carbonate 2,000 mg and vitamin D 500 international units daily D Calcium carbonate 800 mg and vitamin D 100 international units daily

Explanation The Correct Answer is: A Calcium supplementation provides beneficial effects on bone mass throughout postmenopausal life and may reduce fracture rates up to 50%. Postmenopausal women receiving supplemental calcium over a 3-year period in a placebo-controlled, randomized clinical trial had stable total body calcium and BMD in the lumbar spine, femoral neck, and trochanter compared with the placebo group. Recommendations for calcium carbonate are 1,000-1,500 mg once daily orally. Vitamin D increases calcium absorption in the gastrointestinal tract, making calcium more available for reabsorption and circulation. Recommendations for Vitamin D are 800-2,000 international units once daily orally. Calcium Carbonate less than 1,000 mg per day is not sufficient (B and D). Vitamin D levels less than 800 IU per day is not sufficient (C and D).

A 32-year-old female professional golfer presents with a 3-week history of pain along her thumb and down her wrist. She denies any trauma and states that it is aggravated with any movement of her wrist and thumb. Her physical examination is unremarkable. You perform the following test: This test requires her to cup her thumb in a closed fist and ulnar deviate, which reproduces her pain. Considering the suspected diagnosis based on the positive examination above, what are the affected anatomical structures? A Extensor pollicis brevis and abductor pollicis longus B Flexor pollicis longus and flexor digitorum profundus C Extensor carpi radialis longus and brevis D Ulnar collateral ligament

Explanation The Correct Answer is: A De Quervain tenosynovitis is a common condition that occurs in patients who have experienced excessive use of the thumb or wrist. This is a tenosynovitis of the extensor pollicis brevis and abductor pollicis tendons (A), where the tendons lie in the groove of the radial styloid. The diagnosis of De Quervain tenosynovitis is supported by a history of pain in this location along with a painful range of motion of the thumb and further confirmation may be provided by a positive Finkelstein test. The flexor pollicis longus and flexor digitorum profundus (B) are affected in carpal tunnel syndrome. Extensor carpi radialis longus and brevis (C) are extrinsic extensor muscles of the hand. The ulnar collateral ligament (D) is injured in Gamekeeper's thumb.

Patients in which of the following age groups are least likely to experience a dislocation or sprain when a significant stress is placed on their joints? A 5-10 years old B 15-20 years old C 35-40 years old D 55-60 years old E 75-80 years old

Explanation The Correct Answer is: A Dislocations and ligamentous injuries are uncommon in prepubertal children as the ligaments and joints are quite strong as compared to the adjoining growth plates. Excessive force applied to a child's joint is more likely to cause a fracture through the growth plate than a dislocation or sprain. (Roberts, James R., Fractures, Merck Manual Online Medical Library, Merck, Sharp and Dohme Corporation, 2007)

You are getting ready to evaluate a patient who has a past history of gout. The chief complaint is that the patients gout has flared up again, causing pain. Based on your knowledge of gout, which joint is most commonly the site of an initial gout attack? A 1st metatarsaphalangeal joint B Knee C Wrist D Elbow E Ankle

Explanation The Correct Answer is: A Eighty percent of gout attacks affect only one joint. The most common joint to be involved is the 1st metatarsal phalangeal joint. This common phenomenon is called podagra. Gout can affect other joints as well, including the knee, ankle and tarsal joints of the foot. Upper extremities are not commonly affected by gout. The knee is the most common joint to be affected by pseudogout followed by the wrist, MCP joints of the hands, hips, shoulders, elbows and spine. (Sarwark, Ed., Essentials of Musculoskeletal Care, 4 th Edition, 2010)

A 40- year-old female patient comes to the office because she notices that she is easily fatigued but cannot pinpoint a direct cause of her fatigue. She has experienced muscle tenderness to the shoulders and other large muscle groups, sensitivity to touch these areas, and has also felt depressed lately. Based on this history and the vague physical exam findings, what is the most likely diagnosis? A Fibromyalgia B Polyarteritis nodosa C Polymyositis D Scleroderma E Sjogren's

Explanation The Correct Answer is: A Fibromyalgia syndrome (FMS) primarily affects woman between the ages of 20 and 60 and is now the second most common condition seen in rheumatologists' offices behind only rheumatoid arthritis. This condition spares the joints, but causes tender areas throughout the soft tissue as well as generalized pain and fatigue. There are several common trigger points, both anteriorly and posteriorly that help confirm the diagnosis. Polyarteritis nodosa is a systemic necrotizing vasculitis that generally affects medium size muscular arteries. Patients can present with many of the same symptoms as those with fibromyalgia (fatigue, muscle pain, and others), but these patients often have skin manifestations, joint pain, and fever, along with renal, GI, and cardiovascular signs not seen in fibromyalgia. Polymyositis is an inflammatory condition of the muscles that is characterized by proximal muscle weakness, with much less pain than is seen in fibromyalgia. Despite the proximal muscle weakness there is generally not significant atrophy. Sjogren's syndrome does not generally present with any muscular manifestations. It is a chronic inflammatory disorder that affects the salivary and lacrimal glands resulting in dry eyes and a dry mouth. Scleroderma is classically associated with thickened and hardened skin, but it can have internal organ involvement as well affecting many different body systems. When it affects the musculoskeletal system it tends to affect joints and the areas where tendons cross joints. It can cause contractures, pain, and swelling as well as fatigue and weakness. (Sarwark, Ed., Essentials of Musculoskeletal Care, 4 th Edition, 2010)

A 45-year-old obese male presents with pain and swelling to his left knee. It is reddened and tender to touch. He states that this has happened before but the swelling had gone down over the course of several days. Anti-inflammatories have helped some in the past. An analysis of the arthrocentesis fluid reveals needle-shaped crystals. Based on the results of the fluid analysis found within the synovial fluid, what is the most likely diagnosis? A Gout B Lupus C Osteoarthritis D Pseudogout E Rheumatoid arthritis

Explanation The Correct Answer is: A Gout is a condition found in people that have hyperuricemia (but not all people with hyperuricemia will develop the symptoms of gout). Hyperuricemia is defined as an elevated uric acid level in the blood. Uric acid is a product of purine metabolism. Gout occurs when needle-shaped monosodium urate crystals deposit in soft tissue, which leads to pain and inflammation at the deposition site. The diagnosis can be confirmed by analyzing joint aspirate to identify the classic negatively birefringent urate crystals that have a needle-like shape. Pseudogout can present with a similar clinical picture, but an analysis of joint aspirate shows calcium pyrophosphate crystals that are weakly positive, birefringent rhomboid shaped crystals. Lupus, osteoarthritis, and rheumatoid arthritis all can cause joint inflammation and pain, but none would show any significant crystal deposition during an analysis of a synovial fluid aspirate. (Sarwark, Ed., Essentials of Musculoskeletal Care, 4 th Edition, 2010)

A 30-year-old female presents with a year history of chronic aching pain and stiffness throughout her entire body. She denies any trauma, new activities, or change in symptoms throughout the day. Her past medical history is significant for trouble sleeping at night and irritable bowel symptoms. On physical examination, the only finding is pain produced on palpation of the trapezius, medial knee, and lateral epicondyle of the elbow bilaterally. Her vitals are normal. All laboratory work including a thyroid panel, complete metabolic panel, erythrocyte sedimentation rate, and rheumatoid factor are within normal limits. What is the most likely diagnosis based on these findings? A Polymyositis B Fibromyalgia C Polymyalgia rheumatica D Rheumatoid arthritis E Systemic lupus erythematosus

Explanation The Correct Answer is: A Greater than 25,000 ankle sprains happen in the USA every day and the vast majority of those are inversion ankle sprains. The anterior talofibular ligament is the first, and often only, ligament damaged in inversion ankle sprains. As the force of the inversion increases, other lateral ankle ligaments can be involved. When the anterior tibiofibular ligament is involved, this is referred to as a high ankle sprain and such injuries generally have a prolonged recovery time. Calcaneofibular ligaments are generally the second most frequently injured of the lateral ankle ligaments and when injury occurs it is typically in combination with the anterior talofibular ligament. The deltoid ligament is a very strong ligament on the medial aspect of the ankle. Eversion stresses the deltoid ligament, but strong eversion forces are rare and when they do occur, an avulsion fracture of the medial malleolus is more likely than a significant ligament tear. The posterior talofibular ligament is one of the lateral ankle ligaments and can be injured in an inversion injury, but the rate of injury to this ligament lags far behind the anterior talofibular or calcaneofibular ligaments.

A 22-year-old male hurts his right knee while playing football on artificial turf. He states that he planted his foot and went to turn, but his leg didn't turn with his body. He instantly felt a popping sensation in the knee. A few hours later he develops an effusion. Based on the history of the injury, which knee structure was likely injured? A Anterior cruciate ligament (ACL) B Lateral collateral ligament (LCL) C Medial collateral ligament (MCL) D Posterior cruciate ligament (PCL) E Quadriceps tendon

Explanation The Correct Answer is: A Non-impact rotational or hyperextension forces are the most common mechanisms for sustaining a tear of the ACL. One third of patients report hearing an audible popping sound as their ACL tear occurred. Because the ACL is a vascular structure, when it tears a rapid bloody effusion (hemarthrosis) usually develops which effects mobility of the joint. Lateral collateral ligaments are the least likely to be injured as the type of force necessary to cause injury would be a varus stress which is unlikely to occur in typical circumstances. Medial collateral ligament injuries are fairly common and produced by a valgus force that stresses the ligament. This can occur in many sporting events including those in which another competitor might fall on or dive into the lateral aspect of the knee. Trauma to a knee can result in tears of both the ACL and MCL in certain situations. A tear of the quadriceps tendon usually occurs when a person falls on a knee that is partially flexed. As the quadriceps muscle contract to prevent excessive flexion, the force and momentum of the fall may overwhelm the knee extension mechanism and cause the rupture. No such mechanism occurred in our scenario. Posterior cruciate injuries occur when the tibia is driven posterior in relation to the femur as may happen when a car dashboard is driven into the tibias during a major front impact collision. A powerful hyperextension force can result in both ACL and PCL tears (usually in that order). PCL tears are much more uncommon than ACL tears and don't generally occur with basic rotational forces as described in our patient scenario. (Sarwark, Ed., Essentials of Musculoskeletal Care, 4 th Edition, 2010)

You are evaluating a patient with multiple articular complaints that are consistent with osteoarthritis. The patient is a 60-year-old female who complains of pain to her joints which worsen as the day progresses. On examination you notice that the patient has Heberdens nodes. Based on the history and physical exam findings, what structures become progressively damaged and eroded over time? A Articular cartilage B Fascia C Ligaments D Muscles E Tendons

Explanation The Correct Answer is: A Osteoarthritis is a progressive, irreversible disease that leads to loss of articular cartilage in the joints. Osteoarthritis can affect the weight bearing joints in the lower extremities and spine, but can also be commonly found in the hands and other joints. The occurrence and severity of the condition can be affected by history of past trauma to a joint, long-term wear and tear on a joint, as well as obesity, genetics, and progressive aging. The condition can produce pain, swelling, stiffness, decreased range of motion, joint deformity, crepitus and decreased ability to do tasks of daily living. Osteoarthritis does tend to lead to direct damage to tissues like fascia (which is a fibrous connective tissue that surrounds muscles), muscles or tendons (tendons connect muscles to bones). As osteophytes form and joints become distorted, this could have a secondary impact on ligaments that are responsible for providing joint stability, but ligaments are not primarily affected by osteoarthritis. (Sarwark, Ed., Essentials of Musculoskeletal Care, 4 th Edition, 2010)

What age group is most at risk to develop osteoid osteoma? A Adolescents B Elderly C Middle-aged D Neonates E Toddlers

Explanation The Correct Answer is: A Osteoid osteoma is a benign bone forming tumor that usually develops during a patient's second decade of life. This type of tumor is much more common in boys than girls and typically affects the lower extremities (femur and tibia primarily) and spine more than other areas of the body. Patients typically present with gradually progressive bone pain that is worse at night and does not correlate with activity level. The tumor produces high levels of prostaglandins, so symptoms usually improve in 20-25 minutes if the patient takes a medication like ibuprofen, ASA or other NSAIDS that are prostaglandin inhibitors. A lack of improvement in symptoms with these medications should lead health care providers to consider a different diagnosis. The pain of this condition may cause those afflicted in a leg to limp and have swelling, muscle atrophy or contractures and exquisite point tenderness. The condition usually resolves on its own over time, but symptomatic patients may require surgical resection or radioablation of the tumor. (Tochia, Editor, UpToDate, 2010)

Upon testing a patient for function of the hip extensors, which muscle is considered the primary muscle responsible for most extension? A Gluteus maximus B Pectineus C Semimembranosus D Semitendinosus E Vastus lateralis

Explanation The Correct Answer is: A The gluteus maximus is a large muscle that is partially responsible for giving shape to the buttocks. It is the dominant muscle responsible for hip extension. It is easily palpable with a patient in the prone position with buttocks squeezed together or with the hip extended and the knee flexed. The pectineus muscle is considered a secondary hip adductor. The Semimembranosus and Semitendinosus are two of the three hamstring muscles. They are primary movers in knee flexion, but only secondary contributors to hip extension. Vastus lateralis is one of the four quadriceps muscles and plays a role in knee extension, but not hip extension. (Hoppenfeld, Physical Examination of the Spine and Extremities, 1976)Explanation The Correct Answer is: A The gluteus maximus is a large muscle that is partially responsible for giving shape to the buttocks. It is the dominant muscle responsible for hip extension. It is easily palpable with a patient in the prone position with buttocks squeezed together or with the hip extended and the knee flexed. The pectineus muscle is considered a secondary hip adductor. The Semimembranosus and Semitendinosus are two of the three hamstring muscles. They are primary movers in knee flexion, but only secondary contributors to hip extension. Vastus lateralis is one of the four quadriceps muscles and plays a role in knee extension, but not hip extension. (Hoppenfeld, Physical Examination of the Spine and Extremities, 1976)

A 48-year-old woman presents with a chief complaint of gradually progressing difficulty in climbing stairs over the past 3 months. The physical examination shows there is notable proximal muscle weakness of the upper and lower extremities. The remainder of the examination is unremarkable. The laboratory evaluation shows an elevated serum creatinine phosphokinase level, and a muscle biopsy reveals lymphoid inflammatory infiltrates. Which of the following is the appropriate initial treatment of choice in this patient? A prednisone B azathioprine C methotrexate D immunoglobulin E hydrochloroquine

Explanation The Correct Answer is: A The most likely diagnosis in this patient is polymyositis. This is supported by the finding of a gradual progressive proximal muscle weakness and elevation of creatinine phosphokinase level. The finding of lymphoid inflammatory infiltrates on muscle biopsy confirms the diagnosis. Initial treatment of choice in this condition is the use of a corticosteroid (prednisone). Patients who do not respond to prednisone may then benefit from the use of methotrexate or azathioprine. Both intravenous immune globulin and hydroxychloroquine are effective for the treatment of patients with dermatomyositis that is resistant to prednisone therapy. `

A 23-year-old patient who has recently been on a ski trip presents with pain to the right hand after sustaining a fall. It is difficult to move, and there is pain on movement of the first digit. Based on this history what ligament would the patient most likely have injured? A 1st MCP joint ulnar collateral ligament B 2nd MCP joint ulnar collateral ligament C 3rd MCP joint ulnar collateral ligament D 4th MCP joint ulnar collateral ligament E 5th MCP joint ulnar collateral ligament

Explanation The Correct Answer is: A The ulnar collateral ligament at the base of the thumb, or 1st MCP joint, is often injured in forced abduction, such as a fall while skiing or during other sporting activities. An injury to this ligament has traditionally been called Gamekeeper's Thumb, but the origin of this term referred to a more chronic injury sustained by English gamekeepers as a result of the way they killed rabbits using their hands. Any of the MCP joint ulnar collateral ligaments could be injured in a fall if the mechanism of injury creates significant forces on the ligaments, but the 1st MCP joint is far more commonly injured than the others mentioned above.

A 59-year-old woman with a known history of rheumatoid arthritis presents with relatively severe complaints of pain, notable bony deformity of the hands with extra-articular findings of cutaneous nodules, scleritis, and pleurisy. On physical examination, the patient is found to have splenomegaly. Which of the following is the most appropriate laboratory evaluation to order to further evaluate the suspected diagnosis? A complete blood count (CBC) B uric acid C C-reactive protein D antinuclear antibodies E erythrocyte sedimentation rate

Explanation The Correct Answer is: A This patient presentation of known rheumatoid arthritis with severe deformities, extra-articular findings, and splenomegaly is most likely Felty syndrome. Felty syndrome is characterized by the triad of deforming rheumatoid arthritis, splenomegaly, and neutropenia. The appropriate laboratory test to order would be a CBC to evaluate for neutropenia. Uric acid testing is helpful in evaluating gout but is not relevant to this patient presentation. Ordering an erythrocyte sedimentation rate or C-reactive protein is not necessarily helpful in diagnosing Felty syndrome; in an acute inflammatory flare, both would most likely be elevated. Antinuclear antibodies could be present in 20% to 40% of patients but are not diagnostic of Felty syndrome

33-year-old male is brought by rescue to the emergency department after he had a collision on the side of the road on his motorcycle. He is complaining of pain and swelling of the right ankle, with decreased range of motion and pain to the extremity. You decide to order a radiograph of his right ankle (with results shown below). Based on the radiography provided, what is the nature of this injury? A Bimalleolar ankle fracture involving the distal tibia and fibula B Distal fibula fracture only C Distal tibia fracture only D Lisfranc fracture - dislocation E Talus fracture

Explanation The Correct Answer is: A This radiograph clearly shows fractures of both the distal fibula and tibia (see arrows below). Bimalleolar ankle fractures can also present with a clear fibular fracture and signs of deltoid ligament disruption on the medial side (a palpable gap on the medial side, deltoid ligament tenderness or laxity with an eversion-type stress). This type of injury is considered unstable and generally requires surgical management. Choices "B" and "C" were incorrect since there is clear evidence of fractures on both the lateral and medial aspect of the joint pictured. A Lisfranc fracture - dislocation would seem unlikely as the tarsometarsal joints appear intact, but this is not an ideal view to rule out that injury conclusively. This x-ray does not have any evidence of a talus fracture, but additional views would be needed to be sure no such fracture is present. (Sarwark, Ed., Essentials of Musculoskeletal Care, 4 th Edition, 2010)

Which of the following patterns of stiffness is most characteristic of patients with rheumatoid arthritis? A morning stiffness lasting at least 1 hour B exacerbation of joint stiffness with walking C frequent, brief episodes of stiffness after inactivity D stiffness reflected by a major delay in muscle relaxation E stiffness evidenced by increased resistance to passive movement

Explanation The Correct Answer is: B A ganglion cyst is a cystic collection of synovial fluid within a joint or tendon sheath. Treatment options include cyst aspiration and corticosteroid injection (B), or surgical excision. However, none are typically performed in the ED. Surgery is generally effective but ganglion cysts may reoccur. Referral to a hand surgeon is indicated for persistent or recurrent pain or cosmetic deformity. IV antibiotics (A) would not help since ganglion cysts are not known to be infectious. Physical therapy (C) would aggravate the cyst and would not reduce the size typically. Rupturing the cyst (D) would temporarily reduce the size but recurrence is high.

What type of fracture is not related to an acute bony trauma? A Greenstick B Stress C Oblique D Comminuted E Spiral

Explanation The Correct Answer is: B A stress or fatigue fracture is caused by small, repetitive forces that usually involve the metatarsal shafts, the distal tibia, and the femoral neck (though many other bones may be affected). These fractures may not be seen on initial radiographs. A greenstick fracture is an incomplete traumatic fracture with angular deformity seen in children. An oblique fracture is a traumatic fracture with an angulated fracture line. A comminuted fracture is a traumatic fracture in which there are more than two fracture segments. A spiral fracture is a traumatic fracture that has a multiplanar and complex fracture line usually caused by an excessive rotational force on a bone. (Sarwark, Ed., Essentials of Musculoskeletal Care, 4 th Edition, 2010)

The diagnosis of systemic lupus erythematosus (SLE) is supported by a positive initial antibody screen; however, this test is not specific. Which of the following tests is most specific in the diagnostic evaluation of SLE? A gliadin antibody B antibody to double-stranded DNA (anti-dsDNA) C antinuclear antibody (ANA) D anticentromere antibody E antiribosomal P antibody

Explanation The Correct Answer is: B Autoantibody production is the primary immunological abnormality seen in patients with systemic lupus erythematosus (SLE); the antinuclear antibody (ANA) is most characteristic of SLE and seen in 95% of patients with SLE but is not specific for the diagnosis of SLE. A positive ANA can also be found in patients with lupoid hepatitis, scleroderma, rheumatoid arthritis, Sjögren disease, dermatomyositis, and polyarteritis. ANA testing should be employed as the initial screening test in a patient suspected of having SLE. A negative total ANA test is strong evidence against the diagnosis of SLE, whereas a positive test is not confirmatory of the diagnosis. The most specific antibody tests for SLE are antibodies to double-stranded DNA (anti-dsDNAs) and anti-Smith (anti-SM). Although these tests are more specific for SLE, they are less sensitive than the ANA test. Anti-dsDNA is positive in 60% of patients with SLE and anti-SM is positive in 30% of patients. Anti-dsDNA is more likely to reflect disease activity. Gliadin antibody assay is utilized to assess patients with suspected celiac disease. Anticentromere antibody is associated with CREST (calcinosis, Raynaud phenomenon, esophageal dysmotility, sclerodactyly, and telangiectasia) syndrome in scleroderma. Antibodies to ribonucleoprotein are present in patients with a mixture of overlapping rheumatological symptoms known as "mixed connective tissue disease." (

35-year-old male presents with pain and decreased range of motion after sustaining a fall in which the patient tried to grab onto a bar which pulled his entire arm in the process. Given this clinical scenario, at what cervical motor neuron level would the biceps reflex be testing? A C4 B C5 C C6 D C7 E C8

Explanation The Correct Answer is: B C5 is the primary motor neuron being tested in a biceps reflex. C6 contributes to the brachioradialis reflex primarily, but does have a small role in the biceps reflex. The C7 motor neuron is primarily involved in the triceps reflex. C4 and C8 do not contribute to any primary reflexe

A 32-year-old female professional golfer presents with a 3-week history of pain along her thumb and down her wrist. She denies any trauma and states that it is aggravated with any movement of her wrist and thumb. Her physical examination and x-rays are unremarkable. You perform the following test: This test requires her to cup her thumb in a closed fist and ulnar deviate, which reproduces her pain. What is the most likely diagnosis based on this exam finding? A Colle's fracture B De Quervain tenosynovitis C Carpal tunnel syndrome D Ganglion cyst E Gout

Explanation The Correct Answer is: B De Quervain tenosynovitis (B) commonly occurs in patients who have experienced excessive use of the thumb or wrist. Often, no plausible cause can be found. This is a tenosynovitis of the tendons that lie in the groove of the radial styloid. The patient usually presents with pain along the radial aspect of the wrist that may radiate to the thumb or extend into the forearm. The diagnosis of De Quervain tenosynovitis is supported by a history of pain in this location along with a painful range of motion of the thumb and local tenderness over the distal portion of the radial styloid. Further confirmation of the diagnosis may be provided by a positive Finkelstein test, in which the patient grasps the thumb in the palm of the hand and the examiner ulnar deviates the thumb and hand. This stretches the tendons over the radial styloid and produces sharp pain along the involved tendons. A ganglion cyst (D) and carpal tunnel syndrome (C) would not appear on physical examination in this presentation. Gout (E) would be painful and involve more of the joint. A Colle's (A) fracture would show on her x-ray.

When a bone is fractured, there are the 3 stages of healing. What is the proper order in which the healing occurs? A Inflammatory, Remodeling, Reparative B Inflammatory, Reparative, Remodeling C Remodeling, Inflammatory, Reparative D Remodeling, Reparative, Inflammatory E Reparative, Inflammatory, Remodeling

Explanation The Correct Answer is: B Inflammatory changes happen after fractures and this is followed by a reparative phase and ultimately, a remodeling phase. The healing from acute injuries generally starts with inflammation,. Once the integrity of the bone has been restored on the macro level, bone remodeling continues until full healing has occurred. (Sarwark, Ed., Essentials of Musculoskeletal Care, 4 th Edition, 2010)

You are evaluating a 67-year-old female who has history and physical exam findings consistent with osteoarthritis. Based on your knowledge of the disease, which joints are most commonly affected? A Carpometacarpal (CMC) B Distal interphalangeal (DIP) C Metacarpophalangeal (MCP) D Proximal interphalangeal (PIP) E Radiocarpal

Explanation The Correct Answer is: B Osteoarthritis can affect all of the joints mentioned, but generally has the highest prevalence in the DIP joints, especially the second DIP joint. Osteophyte formation at the DIP joints produce enlargements referred to has Heberden's nodes. The first carpometacarpal joint is likely the second most commonly affected joint. This painful and potentially debilitating condition at the base of the thumb can make grasping activities difficult and be exacerbated by prolonged or strenuous use of the thumb. While metacarpophalangeal and proximal interphalangeal joints can definitely be damaged in osteoarthritis, they are the more classic locations for rheumatoid arthritis changes to be manifested. Radiocarpal joints at the wrist are susceptible to osteoarthritis, but not at the same high frequency as the DIP joints. (Sarwark, Ed., Essentials of Musculoskeletal Care, 4 th Edition, 2010)

How many types of primary osteoporosis are there? A 1 B 2 C 3 D 4 E 5

Explanation The Correct Answer is: B Osteoporosis is a condition characterized by low bone mass, which increases the fragility of bones and leads to an increased risk of fracture. Osteoporosis is defined as being either primary or secondary and primary osteoporosis is further broken down into Type 1 and Type 2. Type 1 is related to decreased hormone levels - estrogen in women and testosterone in men, and is sometimes referred to as "postmenopausal osteoporosis." It is six times more common in women and results in loss of trabecular bone. Type 1 primary osteoporosis often presents with vertebral compression fractures or fractures of the distal radius after a fall. Type 2 primary osteoporosis is sometimes referred to as "senile osteoporosis" and generally occurs in patients over 70 years of age. It is twice as common in women as men and occurs due to a diminished capacity to make new bone. The most common types of fractures found in this type of osteoarthritis are hip and pelvic fractures. Secondary osteoporosis occurs at a somewhat higher rate in men versus women and is caused by some other medical condition that produces bone loss. Common causes include long-term steroid use, various endocrine abnormalities, and neoplastic diseases such as multiple myeloma.

A 72-year-old female presents with a 4-month history of pain and stiffness in her shoulders and hips. She identifies the pain being worse in the morning and aggravated with getting in and out of the car along with difficulty brushing her hair. She also reports malaise and a 10-pound weight loss over the past few months. Her blood work shows an erythrocyte sedimentation rate (ESR) of 74 mm/h. What is the best treatment for the suspected diagnosis? A Clindamycin B Prednisone C Acetaminophen D Tramadol E Oxycodone

Explanation The Correct Answer is: B Patients with polymyalgia rheumatica (a clinical diagnosis based on pain and stiffness of the shoulder and pelvic girdle areas, frequently in association with fever, malaise, and weight loss) are treated with prednisone 10-20 mg/day orally. Usually after 2-4 weeks of treatment, slow tapering of the prednisone can be attempted. Most patients require some dose of prednisone for a minimum of approximately 1 year; 6 months is too short in most cases. Clindamycin (A) is an antibiotic that will not help polymyalgia rheumatic. Acetaminophen (C) alone has not been shown as an effective treatment. Tramadol (D) and oxycodone (E) are pain medications that will not help with the specific condition.

What term is used to describe the forward movement of one vertebral body on the vertebra below it, as shown in this figure? A Spina bifida B Spondylolisthesis C Spondylolysis D Sprengel's deformity E Sustentaculum tali

Explanation The Correct Answer is: B Spondylolisthesis is the forward movement of one vertebral body on the vertebra below. This most commonly occurs with L5 on S1 (more than 85% of cases) or L4 on L5. It is often due to a spondylosis (a bony defect) in the pars articularis as pictured above which is an acquired condition that may develop as a result of a stress fracture and it is not uncommon in children or adolescents. A small number of spondylolithesis cases are congenital. Spina bifida refers to a non-union of the vertebral arch at the spinous process, which is a congenital condition. Sprengel's deformity refers to a scapula that only partially descends from the neck to the thorax. The higher than normal position of the scapula results in a shortened appearance of the neck. The sustentaculum tali refers to an anatomical landmark in the foot that supports the talus and serves as an attachment for the spring ligament. It is part of the calcaneus bone.

Which bone is the most susceptible and most often fractured at birth? A Calcaneus B Clavicle C Femur D Humerus E Patella

Explanation The Correct Answer is: B The clavicle is the most common bone broken during childbirth. It often is associated with shoulder dystocia, but clavicular fractures can occur in uncomplicated pregnancies. They are usually of the greenstick variety and heal without complications. Calcaneal and patellar fractures are highly unlikely to occur since they are not long bones which are much more vulnerable to fracture. Fractures of the humerus and femur are possible during childbirth, but generally only in traumatic births. Humerus and femur fractures are much less common than clavicular fractures. (Kendig, Merck Manual Online Medical Library, 2007)

A disk herniation that is putting pressure on the L5 nerve root may present with weakness of what muscle(s)? A Anterior tibialis B Extensor hallucis longus C Gastrocnemius-soleus D Iliopsoas E Peroneus longus and brevis

Explanation The Correct Answer is: B The extensor hallucis longus muscle's motor function is associated the L5 motor neuron, which also supplies the gluteus medius and extensor digitorum longus and brevis muscles. The anterior tibialis muscle is supplied by the L4 motor neuron. Nerves emanating from T12, L1, L2 and L3 supply the iliopsoas. Gastrocnemius, soleus and peroneus longus and brevis are all supplied by nerves coming from the S1 area. The plantar flexing gastrocnemius and soleus muscles also are supplied by S2. (Sarwark, Ed., Essentials of Musculoskeletal Care, 4 th Edition, 2010)

A 5-year-old male is being evaluated for an acute injury to the right ankle. On the x-ray of the ankle there is a distal tibia fracture that involves the separation of the growth plate, as well as a small non-displaced chip fracture of the metaphysis of the tibia. Based on these findings, what type of Salter-Harris fracture does this child have? A I B II C III D IV E V

Explanation The Correct Answer is: B The growth plate is the most fragile part of the bone prior to bone maturation and thus is usually the first structure disrupted when force is applied. Statistically, Type II fractures are most common - those that involve both the growth plate and a chip fracture of the metaphysis

Upon testing a patient for function of the hip flexors, which muscle is considered the primary muscle responsible for most flexion? A Gracilis B Iliopsoas C Rectus femoris D Sartorius E Vastus intermedius

Explanation The Correct Answer is: B The iliopsoas muscle is the primary hip flexor muscle. It originates at T12 and L1-5 vertebrae and intervertebral disks as well as the iliac fossa of the pelvis and connects to the femur at the lesser trochanter. The gracilis muscle is considered a secondary hip adductor. Rectus femoris does help with hip flexion, but in a secondary role to the iliopsoas. Rectus femoris is also involved in knee extension. Sartorius is also involved in hip flexion, but in a secondary role. The vastus intermedius muscle is one of the four quadriceps muscles and is involved with knee extension and is not involved in hip flexion.

Which peripheral nerve is involved in the most common compression neuropathy in the upper extremity? A Axillary B Median C Radial D Sciatic nerve E Ulnar

Explanation The Correct Answer is: B The median nerve is commonly compressed as it passes through the carpal tunnel in the wrist. This syndrome is most often diagnosed in middle aged or pregnant female patients. The axillary nerve passes through the axilla and is often compressed when patients use crutches improperly and bear weight on the axillary area. The ulnar nerve is second only to the median nerve and can be compressed as it passes through the cubital tunnel at the elbow or as it passes through the humeral and ulnar heads of the flexor carpi radialis muscle. Compression of the radial nerve (and its branches) as it passes through the radial tunnel on the lateral side of the elbow is often confused with lateral epicondylitis. The sciatic nerve is associated with the lower extremities and pain is often elicited as a result of a lumbar disk herniation causing nerve root impingement. (Sarwark, Ed., Essentials of Musculoskeletal Care, 4 th Edition, 2010)

The term "nursemaid's elbow" refers to which of the following physical conditions? A A fracture of the humerus B A subluxation of the radial head C Inflammation at the lateral epicondyle D Inflammation of the medial epicondyle E Olecranon bursitis

Explanation The Correct Answer is: B This is the most common elbow injury in children under the age of 5. The injury generally occurs when the child's arm is forcefully pulled when the elbow is in an extended position and the forearm is pronated. Fibers of the annular ligament that encircle the radial neck become trapped between the radius and ulna. On presentation, children hold their arm in slight flexion and pronation. There is no fracture associated with nursemaid's elbow and the olecranon bursa is unaffected. Nursemaid's elbow is an acute injury, while medial and lateral epicondylitis (also known as golfer's elbow and tennis elbow respectively) are generally chronic conditions. (Sarwark, Ed., Essentials of Musculoskeletal Care, 4 th Edition, 2010)

You are evaluating a 53-year-old post-menopausal patient during her routine annual examination in a primary care office. You notice that the patient has not been properly assessed for risk for osteoporosis, and does have some complaints that are of concern for osteoporosis. Based on this history, and the standard of care, what test is used to confirm osteoporosis in this patient? A Bone scan B CT scan C DEXA scan (dual energy x-ray absorptionmetry) D MRI scan E X-ray

Explanation The Correct Answer is: C A DEXA scan is the best test to determine bone density and is commonly used as a screening tool for those at risk of developing osteoporosis. This test is fast, reproducible and exposes patients to a relatively low dose of radiation. The results are reported as Z and T scores. Z scores indicate how the patients bone density compares to peers and T scores compare the patient to young, health individuals. Z and T scores are presented in the form of standard deviations below the comparison group. If the T score is 0 to greater than -1 the test is interpreted as normal. T scores of -1 to -2.5 are indicative of someone with osteopenia and a T score lower than -2.5 is consistent with osteoporosis. None of the other imaging options presented are ideal for determining bone density, but they each play important roles in other aspects of patient care. Bone scans are best suited for imaging studies designed to look for occult fractures, tumors, inflammatory or infectious processes within the bones, or metabolic bone diseases. CT is useful for a detailed examination of bone when looking for fractures or lesions and intra-articular pathology. MRI is helpful in the evaluation of spinal column pathology and various soft tissue injuries involving the muscles, tendons, ligaments, and cartilage. MRI is also useful in the evaluation of stress and occult fractures, osteomyelitis and early osteo necrosis. Radiographs (x-ray) are useful for most initial evaluation of musculoskeletal pathology, but are somewhat limited in their usefulness in visualizing some conditions. Low bone density can be perceived on x-ray, but only after the bone loss has been rather extensive. It is not a great early screening tool for osteoporosis. (Sarwark, Ed., Essentials of Musculoskeletal Care, 4 th Edition, 2010)

A 7-year-old male presents to the emergency department with right leg pain after falling off a swing at the school playground. Imaging of the right lower extremity shows the following fracture pattern: Which type of Salter-Harris Classification is observed? A Type I B Type II C Type III D Type IV E Type V

Explanation The Correct Answer is: C A Salter-Harris Type III fracture (C) involves a portion of the epiphysis only. Type I (A) is the entire epiphysis, Type II (B) is the entire epiphysis along with a portion of the metaphysis, Type IV (D) involves a portion of the epiphysis along with a portion of the metaphysis, and Type V (E) is a compression injury of the epiphyseal plate (nothing is "broken off").

What types of connective tissue are injured in a sprain? A Bones and muscles B Fascia and joint capsules C Ligaments and joint capsules D Muscles and tendons E Tendons and bones

Explanation The Correct Answer is: C A sprain involves injury to those tissues that give support to joints - ligaments and joint capsules. Injury to muscles, tendons, and fascia would all be classified as a strain. Injuries to bone would be classified as a fracture. (Sarwark, Ed., Essentials of Musculoskeletal Care, 4 th Edition, 2010)

A 64-year-old female presents with acute onset of severe right knee pain. She denies any trauma and her history is only significant for diabetes mellitus. On physical examination her knee has significant edema and erythema, with warmth on palpation. After an aspiration of the synovial fluid the results are: Color: opaque yellow fluid WBC: 90,000/mcL PMN: 90% Considering your suspected diagnosis what is the best treatment for this patient? A IV analgesics and observation B IV fluids C Hospitalization and IV antibiotics D PO antibiotics and follow-up in 3 days E MRI for further evaluation

Explanation The Correct Answer is: C Acute bacterial septic arthritis is associated with white blood cell counts commonly > 50,000/mcl with 90% or more polymorphonuclear cells. Compromised immunity, such as diabetes mellitus, increases the risk of septic arthritis. The effective treatment of septic arthritis requires appropriate antibiotic therapy together with drainage of the infected joint. Hospitalization is always necessary to prevent sepsis and administer IV antibiotics (C). IV analgesics (A) can be used but are not a necessary treatment option. IV fluids (B) may help but would not treat the septic joint.

A 7-year-old is diagnosed with an acute case of hematogenous osteomyelitis accompanied with fever and leukocytosis. Based on your knowledge of the disease, which bone is most likely to present with the infection? A Feet B Hands C Long bones D Pelvis E Vertebrae

Explanation The Correct Answer is: C Fortunately, hematogenous osteomyelitis is not common in children, but when it does occur it primarily is found in the long bones. The femur, tibia and humerus are the most typical locations for osteomyelitis in children. The highly vascular metaphysis of long bones contribute to the potential for hematogenous spread of the implicated pathogen. Osteomyelitis can occur at any of the locations mentioned in the answer choices given, but at a significantly lower rate than in the long bones. The rate of occurrence at several selected locations is given below: Feet - 9% Femur 25% Hands - 6% Humerus - 13% Pelvis - 8% Radius/ulna - 6% Tibia/fibula - 28% Vertebrae - 2%

A 58-year-old man with a medical history of gouty arthritis presents with a red, swollen joint at the base of the great toe. His diet for the past 7 to 10 days consisted of large quantities of organ meats and fresh seafood. The increased metabolism of which of the following most likely contributed to the patient's symptoms? A amino acids B polysaccharides C purines D pyrimidines E triglycerides

Explanation The Correct Answer is: C Purines are normally metabolized into uric acid by the liver and can be found in high amounts in several foods, including organ meats, seafood, beans, peas, and many others. Higher levels of meat and seafood consumption are associated with an increased risk of gout because of the hyperuricemia that can occur via purine metabolism. (Hellman and Imboden, 2008, pp. 706-708) Hellman DB , Imboden JB Jr. Arthritis and musculoskeletal disorders. In: McPhee SJ , Papadakis MA, eds. Current Medical Diagnosis & Treatment. New York, NY: McGraw-Hill Medical; 2008.

A 64-year-old postmenopausal female presents for an annual examination. She is 5'0" and weighs 92 pounds; compared to her examination 3 years ago she has lost an inch in height. After performing a dual-energy x-ray absorptiometry (DXA) scan the diagnosis of osteoporosis is confirmed and she is started on appropriate treatment. How often should she be recommended to follow-up with a DXA bone density scan? A Every six months B Every year C Every two years D Every ten years E No need to follow-up

Explanation The Correct Answer is: C Recommendations for patients diagnosed with osteoporosis without a fracture is every 2-3 years (C). This guideline is approved based on age, risk factors, or previous fractures. A post-menopausal woman at age 64 without a presenting fracture can be followed every 2-3 years with bone density imaging (DXA scan). Medications and other treatment options may need earlier or more frequent follow-up depending on the treatment plan. DXA is quite accurate and delivers negligible radiation. Performing a DXA at intervals less than two years has not shown of benefit (A and B). Every 10 years is too far of a time frame to follow osteoporosis (D). Follow-up is needed to see any changes in the bone density (E).

Radiculopathy due to nerve root compression occurs most commonly at which nerve root within the brachial plexus? A C5 B C6 C C7 D C8 E T1

Explanation The Correct Answer is: C The C7 nerve root is affected the most often (approximately 45-60%). This radiculopathy can result from foraminal encroachment of the spinal nerve, cervical disk herniation, tumor, and multiple sclerosis. C7 radiculopathy can present with weakness in the triceps, which cause elbow extension, and finger flexion and extension. C6 is another common site of radiculopathy. C6 radiculopathy can present with weakness in the biceps, brachioradialis, and wrist extensor muscles. Cervical radiculopathy at the C5, C8, and T1 are less common, but still possible. C5 radiculopathy can present with deltoid and biceps muscle weakness. C8 radiculopathy can present with finger flexor weakness and T1 radiculopathy with finger abduction weakness.

A 32-year-old female professional golfer presents with a 3-week history of pain along her thumb and down her wrist. She denies any trauma and states that it is aggravated with any movement of her wrist and thumb. Her physical examination is unremarkable. Considering the suspected diagnosis you perform the following examination: This test requires her to cup her thumb in a closed fist and ulnar deviate, which reproduces her pain. Which test was performed? A Apley's B Phalen's C Finkelstein's D Tinel's E Empty Can

Explanation The Correct Answer is: C The diagnosis of De Quervain tenosynovitis is supported by a history of pain in the location of the radial aspect of the wrist with a painful range of motion of the thumb and occasional local tenderness over the distal portion of the radial styloid. Further confirmation of the diagnosis may be provided by a positive Finkelstein test (C), in which the patient grasps the thumb in the palm of the hand and the examiner ulnar deviates the thumb and hand. This stretches the tendons over the radial styloid and produces sharp pain along the involved tendons. A Tinel's (D) and Phalen's (B) sign are positive in carpal tunnel syndrome. The Apley's compression test (A) is used to evaluate meniscus tears in the knee. The Empty Can Test (E) is used to assess the rotator cuff supraspinatus muscle.

Upon testing a patient for function of the hip abductors, which muscle is considered the primary muscle responsible for most abduction? A Biceps femoris B Gluteus maximus C Gluteus medius D Gluteus minimus E Vastus medialis

Explanation The Correct Answer is: C The primary mover in the motion of hip abduction is the gluteus medius muscle. Gluteus minimus does play a supporting role in that motion. Biceps femoris is one of the three hamstring muscles and contributes to the motions of knee flexion (primary muscle) and hip extension (secondary muscle). Gluteus maximus is the primary mover for hip extension and vastus medialis is one of the four quadriceps muscles responsible for knee extension, but no hip movements. (Hoppenfeld, Physical Examination of the Spine and Extremities, 1976)

A 78-year-old female trips and falls on an outstretched hand and now presents with right wrist pain. On examination, the patient complains of pain on palpation to the right radial side of the wrist near the anatomical snuffbox. Based on this clinical presentation, what is the most likely fracture that this patient has? A Capitate B Lunate C Scaphoid D Trapezium E Trapezoid

Explanation The Correct Answer is: C The scaphoid or navicular bone of the wrist, is the most commonly fractured carpal bone. This injury occurs most commonly in young men. The lunate is the second most commonly fractured carpal bone and the most commonly dislocated carpal bone. The capitate is the largest of the carpal bones and it is not known as a common isolated fracture site. Fractures of the trapezium and trapezoid are relatively uncommon as well.

A 27-year-old female presents to the emergency department after a motor vehicle accident. Imaging of the left lower extremity shows the following fracture pattern: How would this fracture be described? A No displacement B Complete dorsal displacement C Fifty percent dorsal displacement D Displacement with shortening

Explanation The Correct Answer is: C This Fracture is displaced, pertaining to any deviation from anatomical position or alignment to the extent to which the fracture fragments are nonconcentric or offset from each other. This fracture is only 50 percent displaced without angulation or shortening. The magnitude of displacement is expressed in either terms of measurement (i.e. incomplete) or percentage (i.e. 50%) ) of the width of the bone. The direction of displacement is based on the position of the distal fragment relative to the proximal fragment. Separation is the distance two fragments have been pulled apart. Shortening (D) is the amount the bone's length has been reduced, which is not seen in this fracture. Angulation is the degree of "bending" that makes the fragments unparalleled. This would not be described as "no displacement" (A) or "complete displacement" (B).

An avulsion fracture at the base of the fifth metatarsal is commonly called which of the following? A Bennett fracture B Boxer's fracture C Chauffer's fracture D Jones fracture E Lisfranc fracture

Explanation The Correct Answer is: D An avulsion fracture at the base of the fifth metatarsal, usually secondary to plantar flexion and inversion is called a Jones fracture. Also called a ballet or dancer's fracture, it is the most common metatarsal fracture. The fracture occurs at the proximal diaphysis. A Bennett fracture is an oblique fracture of the first metacarpal near the carpometacarpal joint. A boxer's fracture is a fracture of the fifth metacarpal. This is the most common fracture of the hand. A chauffer's fracture is an oblique fracture through the base of the radial styloid in the forearm. A Lisfranc fracture is actually a fracture and dislocation involving the tarsometatarsal joints.

A 22-year-old man presents with an insidious onset of low back pain over the last 6 months. He describes the pain as dull and has difficulty localizing the pain. The pain often radiates to his thighs. The pain is worse in the morning and associated with stiffening that lessens during the day. The patient notes that there is no history of trauma. The initial laboratory evaluation shows an elevated erythrocyte sedimentation rate, positive HLA-B27, and a negative rheumatoid factor. Plain films of the lumbar spine reveal bilateral blurring of the sacroiliac joints. Which of the following is the most likely diagnosis? A systemic lupus B lumbar disc disease C rheumatoid arthritis D ankylosing spondylitis E polymyalgia rheumatica

Explanation The Correct Answer is: D Ankylosing spondylosis is the most likely diagnosis in this patient. This condition is a chronic inflammatory disorder of the joints of the axial skeleton and commonly presents in the late teens or twenties. Male patients have a higher incidence than do female patients. A common presentation is pain in the lower back with radiation to the thighs and associated limitation of movement that may lessen during the day. Laboratory findings include an elevated erythrocyte sedimentation rate and positive HLA-B27. The HLA-B27 is not a specific test for ankylosing spondylitis; a small percentage of the normal population has a positive finding of this antigen. The earliest radiographic findings occur in the sacroiliac joints, with the detection of erosion and blurring of the joint space. Systemic lupus commonly affects women of childbearing years and presents with exacerbations and remissions of arthritis, rash, fatigue, and the potential for organ system involvement. Lumbar disc disease is usually seen in the age group of 35 to 45 years and is more likely to be associated with trauma. Rheumatoid arthritis does have the potential to affect this age group, but it would more likely be associated with smaller joints of the hands, along with a positive rheumatoid factor. Polymyalgia rheumatica more commonly affects patients older than 50 years and is associated with fatigue, malaise, chronic pain, and stiffness of the proximal muscles, shoulders, neck, and pelvic girdle.

A 16-year-old male soccer player is complaining of pain to the right foot that has been getting progressively worse for the last 2 months. He states it hurts the most when he has all of his weight on his right foot as he plants to kick the ball. Most of the pain appears to be on weight bearing. You are concerned that this patient may be developing a stress fracture. Based on the patient's history and patient presentations, which bone is the most affected by stress fractures in the foot? A Calcaneus B Fifth metatarsal C First metatarsal D Second metatarsal E Talus

Explanation The Correct Answer is: D Any bone that is exposed to repetitive stress can have a stress fracture, but the long and thin metatarsal bones of the foot are the most commonly affected bones. Of the metatarsals, the second metatarsal has the highest number of stress fractures. These weight bearing bones can be particularly vulnerable to stress fracture if the patient is involved in long distance running, especially if he/she is wearing improper footwear for that activity or footwear that has lost most of its shock absorbing abilities. Some young female athletes may be training so hard that they become amenorrheic which can contribute to osteopenia resulting in weaker bones. Older patients with osteoporosis will also have a higher risk of stress fracture. Initially stress fractures of the metatarsals may present with a small area of localized pain and the dorsal forefoot may demonstrate a fairly diffuse area of swelling. If the stress fracture is not treated early, some patients will experience an audible pop or crack as the incomplete stress fractures progresses to a complete break. All types of fractures occur more easily in long thin bones like the metatarsals, than thicker bones like the calcaneus and talus.

What nerve is most commonly injured in a mid- or distal humeral shaft fracture? A Axillary B Median C Peroneal D Radial E Ulnar

Explanation The Correct Answer is: D Because of the radial nerves proximity to the humerus, mid and distal shaft fractures with significant displacement can cause a radial nerve injury. Median and ulnar injuries are more commonly associated with forearm injuries. Axillary nerve injuries are most common in anterior shoulder dislocations and peroneal nerve damage occurs as a result of lower leg insult. (Tuncali, et. al., "Upper Extremity Nerve Injuries: The Significance of Soft Tissue Associations," Neuroanatomy, Volume 3, 2004)

A 17-year-old female distance runner with no significant PMH complains that she has diffuse, aching anterior knee pain that is worsened when she walks up or down stairs or when she squats down. There has been no acute trauma, but she has been increasing her running mileage. No effusion is present. What is the probable diagnosis? A Anterior cruciate ligament tear B Medial meniscal tear C Osteoarthritis of the knee joint D Patellofemoral syndrome E Posterior cruciate ligament tear

Explanation The Correct Answer is: D Patellofemoral syndrome is a common condition in active adolescents due to repetitive stresses on the patellofemoral joint. This can be exacerbated by altered patellofemoral tracking due to growth and development in adolescents. Other biomechanical issues that may contribute to this problem include poor flexibility, weakness of the vastus medialis muscle, which results in a more lateral tracking of the patella, and excessive foot pronation. This condition does not produce an effusion. Anterior and posterior ligament tears would be highly unlikely given that there was no history of an acutely traumatic event and because ligaments are fairly vascular structures, an effusion would be expected with any cruciate ligament tear. Meniscal tears in a young patient are generally associated with a traumatic twisting event and while not as vascular as a ligament, meniscal tears generally lead to the gradual development of an effusion. Older patients may experience a degenerative tear with minimal trauma, but that is unlikely in our 17-year-old patient. Osteoarthritis is generally associated with gradual wear and tear after many years or the process can be accelerated after a significant acute trauma to a joint. Neither scenario is applicable to this patient. (

Tumor necrosis factor (TNF) inhibitors are most often considered for use in patients with rheumatoid arthritis (RA) that does not respond to initial therapy. Which of the following screenings should occur before a patient is placed on this class of medication? A chest x-ray B allergy testing C liver function tests D purified protein derivative (PPD) test E serum BUN (blood urea nitrogen) and creatinine test

Explanation The Correct Answer is: D Patients being treated for rheumatoid arthritis with tumor necrosis factor (TNF) inhibitors are at increased risk for developing an opportunistic infection, such as tuberculosis (TB). It is recommended that screening for the presence of latent TB occur before TNF inhibitors are started. There is no specific indication to order a chest x-ray, allergy testing, liver function tests, or serum BUN and creatinine prior to initiation of TNF inhibitors. (Furst et al, 2009, pp. 633-635; Hellmann and Imboden, 2008, p. 734) Furst DE , Ulrich RW , Varkey-Altamirano C. Nonsteroidal anti-inflammatory drugs, disease-modifying antirheumatic drugs, nonopioid analgesics, and drugs used in gout. In: Katzung BG , Masters SB , Trevor AJ, eds. Basic and Clinical Pharmacology. New York, NY: McGraw-Hill; 2009:621-642. Hellmann DB , Imboden JB. Arthritis and musculoskeletal disorders. In: Tierney LM , McPhee SJ , Papadakis MA, eds. 2008 Current Medical Diagnosis and Treatment. New York, NY: McGraw-Hill; 2008:703-756.

A 72-year-old female presents with a 4-month history of pain and stiffness in her shoulders and hips. She identifies the pain being worse in the morning and aggravated with getting in and out of the car along with difficulty brushing her hair. She also reports malaise and a 10-pound weight loss over the past few months. Her blood work shows an erythrocyte sedimentation rate (ESR) of 74 mm/h. What is the most likely diagnosis? A Gouty arthritis B Bursitis C Fibromyalgia D Polymyalgia rheumatica E Septic arthritis

Explanation The Correct Answer is: D Polymyalgia rheumatica (D) is a clinical diagnosis based on pain and stiffness of the shoulder and pelvic girdle areas, frequently in association with fever, malaise, and weight loss. Because of the stiffness and pain in the shoulders, hips, and lower back, patients have trouble combing their hair, putting on a coat, or rising from a chair. Elevated ESRs are present in most cases being > 50 mm/h. The age group most affected is over 50 years old. Fibromyalgia (C) is not specific for only the shoulders and hips. Bursitis (B) would usually be specific to one location. Gouty arthritis (A) is usually monoarticular and is not commonly seen in the hips and shoulders on presentation. Septic arthritis (E) would be joint specific and more acute.

A 65-year-old male presents with pain and swelling to his right knee without any history of injury. He has had this type of pain and swelling before, and does recall that he had fluid drained out of the knee several years ago. On examination the patient has a swollen, tender knee with a palpable effusion. There is decreased range of motion to the joint secondary to the effusion. An arthocentesis is performed, and the analysis of the fluid reveals calcium pyrophosphonate crystals. Based on these findings, what is the most likely diagnosis? A Septic arthritis B Acute synovitis C Hemarthoma D Pseudogout E Occult fracture

Explanation The Correct Answer is: D Pseudogout is also known as Calcium Pyrophosphate Deposition Disease (CPDD) and most commonly affects patients over the age of 65. The knee joint is most commonly affected with the wrist joint second most common. An examination of synovial fluid aspirated from the affect joints, will reveal calcium pyrophosphate crystal. Ankle joints are rarely affected by pseudogout but are fairly common locations for gout caused by urate crystal deposition. The first metatarsal phalangeal joint is the classic location for gout (sometimes referred to as podagra), but it is not a typical pseudogout location. The glenohumeral joint of the shoulder is affected by pseudogout much more than gout. Distal interphalangeal joints of the hand are rare locations for either gout or pseudogout, but are classic locations to see signs and symptoms of osteoarthritis. When pseudogout does affect the hands it is generally seen in the metacarpophalangeal joints. (Sarwark, Ed., Essentials of Musculoskeletal Care, 4 th Edition, 2010)

A 56-year-old male who works in construction climbing ladders has developed pain to the right foot for several days. You have seen and examined the patient a few days after the patient started complaining of pain to the foot. Your initial x-rays of the foot are negative for fracture. He continues to have pain, and decreased ability to bear weight. Based on this clinical scenario, how many days after the initial examination should another x-ray be ordered to look for a stress fracture? A 1-2 days B 5-7 days C 10-12 days D 14-30 days E 180 days

Explanation The Correct Answer is: D Radiographic evidence of stress fractures is not immediately apparent after the initial onset of symptoms. Estimates of anywhere from 14 to 42 days before visible signs of a stress fracture can be detected on X-ray have been put forth with a commonly utilized range of 14 to 30 days. Any answer choice above that included numbers less than 14 days would simply be inaccurate due to the gradual appearance of the classic x-ray findings of a stress fracture. After 180 days from the time of stress fracture onset, the fracture should be nearly healed if managed properly. If a stress fracture is highly suspected in light of negative x-rays, imaging with a bone scan should help make the diagnosis. MRI scans may help confirm the diagnosis, but they are rarely utilized in the work-up of suspected stress fractures. (Sarwark, Ed., Essentials of Musculoskeletal Care, 4 th Edition, 2010)

A 72-year-old female presents with a several month history of symmetric pain to the hands, wrists, and feet. She also is noticing a change to the appearance of the joints of the hands and feet, with a more "nobby" appearance than before. When the patient wakes up in the morning, the joints are particularly stiff and red at times. Out of all the areas that are painful, it is the hands that are the most painful. Based on this history, what physical exam findings would you expect to find on the hands that are classic findings of this disease? A Radial deviation and subluxation of the DIP joints B Radial deviation and subluxation of the MCP joints C Radial deviation and subluxation at the PIP joints D Ulnar deviation and subluxation at the MCP joints E Ulnar deviation and subluxation at the PIP joints

Explanation The Correct Answer is: D Rheumatoid arthritis is a systemic autoimmune disease that is incurable and affects woman more than men by a 3 to one ratio. It can damage any joint, but it most often affects joints of the hands, wrists, knees, feet and ankles. The classic appearance of hands that have been afflicted with progressive rheumatoid arthritis is that they demonstrate ulnar deviation and subluxation at the MCP joints. Involvement of the PIP joints is not uncommon, but DIP joints are generally not as significantly affected. Radial deviation is not a typical characteristic of rheumatoid arthritis of the hands. (Sarwark, Ed., Essentials of Musculoskeletal Care, 4 th Edition, 2010)

You are evaluating a 32-year-old male who has possibly injured his Achilles tendon while planting his foot into the soft ground while playing touch football. When evaluating a patient with this suspected injury, you have the patient plantar flex his ankle as part of the exam. Based on your knowledge of anatomy, what two plantar flexing muscles attach into the Achilles tendon? A Anterior tibialis and soleus B Flexor hallucis longus and flexor digitorum C Flexor hallucis longus and gastrocnemius D Gastrocnemius and soleus E Peroneus longus and peroneus brevis

Explanation The Correct Answer is: D The gastrocnemius and soleus muscles form a common tendon at their distal insertions known as the Achilles tendon. This the strongest and thickest tendon in the body and it attaches at its distal end onto the calcaneus bone of the foot. Contraction of the gastrocnemius and soleus muscles are responsible for plantar flexion of the foot and ankle. The anterior tibialis muscle is one of the important dorsiflexors of the foot and ankle. Flexor hallucis longus and flexor digitorum are both involved in plantar flexion, but are not as strong as the gastrocnemius and soleus muscles and do not attach to the Achilles tendon. The flexor hallucis longus is primarily responsible for plantar flexion of the great toe and flexor digitorum produces plantar flexion of the remaining toes. The peroneus longus and peroneus brevis muscles do contribute to plantar flexion, but are not connected to the Achilles tendon. They are also the primary evertor muscles of the foot and ankle. (Hoppenfeld, Physical Examination of the Spine and Extremities, 1976)

22-year-old male presents to the Emergency Department with severe pain in his right arm and is in distress without relief after being given narcotic pain medication. He had a right short arm cast placed for a fractured distal radius 12 hours ago. His pain began 4 hours ago, without any associated trauma. On removal of the cast his arm is noticeably swollen, firm, and tender to any palpation or movement. Based on high clinical suspicion of the suspected diagnosis what is the most appropriate treatment option? A Apply a short arm splint B Emergent MRI for evaluation C IV antibiotics D Emergent surgical consult for a fasciotomy E IV fluids and narcotics with observation

Explanation The Correct Answer is: D The key to early detection of compartment syndrome is a high index of clinical suspicion. Initiate medical management and an emergent surgical consult while the surgical fasciotomy is being arranged. This includes supplemental oxygen and supporting the blood pressure in the hypotensive patient. Place the affected limb at the level of the heart; elevation higher than the heart reduces the arteriovenous pressure gradient. Associated open or closed fractures are common. The patient with compartment syndrome will initially complain of severe pain, often difficult to control even with narcotic pain medications. The pain typically starts within a few hours of the injury, but may develop up to 48 hours after the event. Clinically, the compartment is commonly swollen, firm, and tender to squeezing by the examiner. Removing constrictive casts or dressings, you would not want to reapply a cast (A). An MRI (B) would take too long when time is critical. IV antibiotics would not correct compartment syndrome (C). IV fluids and pain medication (E) can be given while waiting for surgery but are not used to treat compartment syndrome.

A 12-year-old girl presents with complaints of intermittent pain and stiffness involving her hands. This pain has been progressively worsening over the past 3 years. She relates that for the past 2 months she has been feeling increasingly tired and has experienced swelling and stiffness of her hands, which appears worse in the morning and is relieved as the day progresses. The physical examination shows she has a low-grade fever. There are multiple symmetrical joint swelling of the proximal interphalangeal and metacarpophalangeal joints with associated warmth, tenderness, and effusion. Initial laboratory findings include a CBC that reveals mild anemia, an elevated erythrocyte sedimentation rate, a positive rheumatoid factor, and a negative antinuclear antibody (ANA) test. X-rays of the hands and wrists show soft tissue swelling and periarticular osteopenia. Which of the following is the most likely diagnosis? A reactive arthritis B infectious arthritis C systemic juvenile rheumatoid arthritis D polyarticular juvenile rheumatoid arthritis E pauciarticular juvenile rheumatoid arthritis

Explanation The Correct Answer is: D The most likely diagnosis in this patient is polyarticular juvenile rheumatoid arthritis (JRA). This form of JRA is seen in approximately 35% of patients with JRA. It is characterized by symmetrical involvement of five or more joints. Two subsets of the disease exist that are distinguished by the presence or absence of rheumatoid factor. A positive rheumatoid factor is most commonly seen in girls with later disease onset (at least 8 years old). An antinuclear antibody (ANA) test may be positive but is more likely to be positive with the pauciarticular form. In the early stage of the disease, the x-ray may be normal or show soft tissue swelling and periarticular osteopenia. In addition to the positive ANA of pauciarticular JRA patients, the arthritis must be present in four or fewer joints. Early onset disease is commonly seen in girls aged 1 to 5 years and has a positive ANA; up to 30% of patients will also have eye involvement. Late onset disease is more common in male patients, with involvement of the large joints. Systemic JRA, also known as "Still disease," is seen in about 10% to 15% of children with JRA. It is characterized by daily intermittent fever spikes and a transient, nonpruritic, pale pink, blanching macular, or maculopapular rash found on the trunk. A positive rheumatoid factor is rare in this form of JRA. Reactive arthritis is usually associated with a recent viral or bacterial infection. Infectious arthritis more commonly presents as monarticular and is usually acute in onset.

A 56-year-old right hand dominant male presents with swelling to the right elbow. He explains to you that he has had this type of swelling before and was diagnosed with bursitis. Based on this description, where would the most likely site for the elbow bursitis be located? A Capitellum B Lateral epicondyle C Medial epicondyle D Olecranon E Radial head

Explanation The Correct Answer is: D The olecranon bursa is very vulnerable to injury and chronic inflammation due to its superficial location on the extensor side of the elbow. Acute falls, chronic pressure, and other inflammatory processes such as rheumatoid arthritis or gout may contribute to olecranon bursitis. The medial and lateral epicondyles are known to be associated with chronic tendonopathies. The radial head is a common location for subluxation or fracture, while the capitellum at the distal end of the humerus is sometimes associated with fractures and articular cartilage damage. (Hoppenfeld, Physical Examination of the Spine and Extremities, 1976)

39-year-old male patient presents with low back pain with radiation to the right leg. On examination you place the right hip in a flexed position, and as you palpate between the iscial tuberosity and the greater tronchanter of the femur the patient complains of radiation of pain down his right leg. Based on this history and exam, which nerve was affected by this part of the examination maneuver? A Femoral B Peroneal C Saphenous D Sciatic E Sural

Explanation The Correct Answer is: D The sciatic nerve does lie midway between the ischial tuberosity and greater trochanter and it can be palpated when the patient is in a hip flexed position. The gluteus maximus obscures the nerve from being effectively palpated when the leg is in an extended position. Tenderness of the sciatic nerve can be caused by a lumbar disk herniation, direct trauma, or spasm of the nearby pyriformis muscle. The femoral nerve is a deep structure that lies lateral to the femoral artery and is not considered to be palpable. The femoral nerve is responsible for the L1-3 dermatomes and for supplying motor function to the iliopsoas muscle. The peroneal nerve originates from the sciatic nerve and splits into the superficial and deep peroneal nerves, which are responsible for much of the sensory and motor nerve function in the lower leg. The saphenous nerve originates from the femoral nerve in the femoral triangle and runs down the medial aspect of the leg. The sural nerve has medial and lateral components that are found in the lower leg. The medial cutaneous sural nerve arises from the tibial nerve just below the knee and eventually connects with peroneal nerve to form the sural nerve. On the lateral side of the lower leg, the sural nerve arises from the common peroneal nerve just above the knee and eventual connects with the previously discussed medial branch to form the sural nerve. (Hoppenfeld, Physical Examination of the Spine and Extremities, 1976)

A 88-year-old female is found lying on the ground in a supine position after sustaining a fall in her house from tripping on the rug.. Her leg is shortened, abducted and externally rotated, and she is complaining of pain to the right leg with radiation to the knee. Based on the history of the patient, what type of injury has this patient sustained? A Non-displaced pelvic fracture B Hip dislocation C Femoral shaft fracture D Femoral neck fracture E Distal femur fracture

Explanation The Correct Answer is: D When a fracture occurs in the femoral neck that includes displacement, the leg appears shortened, abducted and externally rotated. The tension that exists from muscles attached above and below the fracture site results in shortening of leg length after the fracture as the muscles contract. If there is no displacement of the bone, no leg length changes should occur. Generally, hip fractures would not result in a lengthening of the leg, so all answers that include this option would be wrong. When the leg appears shortened, flexed, adducted, and internally rotated, a posterior hip dislocation should be suspected. A leg that is flexed, abducted and externally rotated suggests an anterior hip dislocation. Posterior hip dislocations are far more common than anterior dislocations, accounting for more than 90% of all hip dislocations.

A 9-year-old female presents to the emergency department after falling out of a second story window. Imaging of the left upper extremity shows the following fracture pattern: How would this fracture be described? A Transverse B Oblique C Spiral D Segmental E Greenstick

Explanation The Correct Answer is: E A greenstick fracture occurs almost exclusively in young children as a result of their bones being more pliable than those of adults. On imaging, a greenstick fracture has cortical disruption on the convex side of the bone with an intact periosteum on the concave side of the fracture. Spiral fractures (C) are often seen in non-accidental or child abuse cases. Transverse (A), segmental (D), and oblique (B) fractures involve both cortices.

A fracture involving the medial epicondyle will most likely cause damage to which nerve? A Axillary B Median C Peroneal D Radial E Ulnar

Explanation The Correct Answer is: E Because the ulnar nerve passes through the cubital tunnel, which is a groove on the posterior aspect of the medial epicondyle, any fractures involving the medial epicondyle can also cause damage to the ulnar nerve. The median nerve is most susceptible to injury at the carpal tunnel. Branches of the radial nerve can become entrapped on the lateral side of the elbow and the associated symptoms are often confused with lateral epicondylitis. Radial nerve injuries are more commonly associated with humeral shaft fractures. The axillary nerve is significantly proximal to the medial epicondyle and the peroneal nerve is in the leg. (Sarwark, Ed., Essentials of Musculoskeletal Care, 4 th Edition, 2010)

A 21-year-old female patient presents with a large area of swelling localized to the front of the knee, between the patella and the skin after a fall directly on her knee. She has been stating that her knee is becoming more difficult to move due to the swelling and pain. She is otherwise healthy with no other medical problems. Her exam reveals a tender, fluctuant area just anterior to the patella without warmth. She does not appear to have any effusion, and her ligament stability testing is normal. Based on these findings, what is the most likely diagnosis? A Anterior cruciate ligament tear B Medial meniscal tear C Pes anserine bursitis D Posterior cruciate ligament tear E Prepatellar bursitis

Explanation The Correct Answer is: E Bursae are synovial fluid filled sacs that facilitate the reduction of friction between adjacent structures. They may be found between the skin and various bony prominences or between tendons, ligaments and bone. They can become injured by an acute direct impact or gradual repetitive stress such as what might occur if someone was required to do extensive kneeling on the job. They may also be sites of infection with staphylococcus aureus and streptococcus species being the most common pathogens. The knee contains two primary bursae. The prepatellar bursa lies directly below the skin and above the patella. It is often acutely injured with a fall or other traumatic impact. A localized swelling can occur quickly and this fluid accumulation is not intracapsular as occurs with a joint effusion after an anterior cruciate ligament tear, meniscal tear or posterior cruciate tear. Meniscal injuries tend to produce a smaller effusion than cruciate ligament tears. The mechanism of falling on a bent knee can be consistent with a posterior cruciate ligament injury, but given the lack of an effusion and the presence of a well contained anterior knee swelling, this becomes less likely. Pes anserine bursa are located on the anterior medial aspect of the tibia, just below the tibial plateau. This location is the insertion of the gracilis, sartorius, and semitendinosus muscles. Pes anserine bursitis is commonly associated with early osteoarthritis in the medial compartment of the knee.

A 52-year-old man recently underwent surgery for a hip replacement. Upon discharge, he is prescribed oxycodone 7.5 mg and acetaminophen 325 mg and told to take 1 to 2 tablets every 6 hours prn to help manage the pain he is expected to encounter as he recovers at home. Which of the following medications would you also recommend for the patient to help minimize potential side effects associated with his pain medication? A esomeprazole B diphenhydramine C guaifenesin D hydrocortisone 1% cream E senna & docusate

Explanation The Correct Answer is: E Constipation is a common adverse effect of opioid therapy. In order to minimize or prevent constipation, the use of stool softeners (docusate) and a stimulant laxative (senna) can be initiated when opioid therapy is begun. (Rabow and Pantilat, 2008, pp. 69, 78) Rabow MW , Pantilat SZ. Palliative care & pain management. In: Tierney LM Jr , McPhee SJ , Papadakis MA, eds. Current Medical Diagnosis & Treatment. 47th ed. New York: McGraw-Hill; 2008.

Approximately what percentage of patients with a solid primary tumor elsewhere will end up with metastatic disease of the vertebrae during the clinical course of their cancer? A 10% B 20% C 30% D 40% E 50%

Explanation The Correct Answer is: E Fifty percent of cancer patients will develop metastatic disease of the vertebrae at some point during the course of their illness. The highest percentages of cancers that lead to such spinal lesions are carcinomas of the breast, lung, prostate, colon, thyroid and kidney. This likely occurs through hematogenous spread. Pain is a common presenting symptom, but it may be found while still asymptomatic if routine screenings are done with bone scans, MRI or CT in patients with a known primary tumor elsewhere in the body. The pain is usually worse with weight bearing activities and better when lying down, but pain that persists through the night and prevents sleep needs to be evaluated for possible neoplasm. Some metastatic disease of the spine may present with neurological symptoms such as sensory or motor deficits following a spinal nerve root distribution or more generalized neurological deficits due to spinal cord compression or cauda equina syndrome. (Sarwark, Ed., Essentials of Musculoskeletal Care, 4 th Edition, 2010)

A 56-year-old insulin dependent diabetic has been under your evaluation for his diabetes for several years. The patient has a 3-year history of diabetic neuropathy to the right foot, and may have suffered an injury to the foot without knowing due to loss of sensation. The patient now presents with a tender, reddened, and swollen right foot for the last 10 days that is also warm to the touch. You suspect that this patient may have an acute case of osteomyelitis. Based on this history, what bacterial organism is most commonly the cause of osteomyelitis? A Group A beta-hemolytic streptococci B Hemophilus influenzae C Mycoplasma D Pseudomonas aeruginosa E Staphylococcus aureus

Explanation The Correct Answer is: E Osteomyelitis is an infection in a bone and can occur in patients of all ages. The most common organism implicated in osteomyelitis across all age groups is Staphylococcus aureus. It can enter the bone through multiple mechanisms including by direct inoculation during an open fracture or during surgical intervention following a fracture (most common mechanisms for adults) or by hematogenous spread from another source (the usual cause in children). Hemophilus influenzae was a much more common organism in pediatric osteomyelitis in the past, but its prevalence is decreasing due to routine immunizations. Group A beta-hemolytic streptococci is the second most common organism found in osteomyelitis in children, while Pseudomonas aeruginosa is the second most common pathogen in adults. Mycoplasma induced osteomyelitis is relatively rare and usually confined to immunocompromised patients.

Which of the following is a type of primary malignant tumor of the bone? A Enchondroma B Leiomyoma C Lipoma D Melanoma E Osteosarcoma

Explanation The Correct Answer is: E Osteosarcoma is a type of primary bone cancer. Fortunately this is a rare cancer, accounting for only 1% of the cancers diagnosed each year in the USA for patients of all ages. Osteosarcomas are more common in the pediatric population, accounting for 5% of all childhood cancers and 56% of all cancers of the bone in patients less than 20 years old. Osteosarcomas have a bimodal occurrence rate with the highest prevalence between ages 11-13 and over the age of 65. Enchondromas are common benign cartilaginous tumors that develop in the medulla (marrow cavity) of bone. A leiomyoma is another name for a benign uterine fibroid. A lipoma is a benign, soft, freely movable, generally nontender mass in the soft tissue sometimes referred to as a fatty tumor. These are generally inconsequential, but may be a marker for spina bifida if found in the lumbar region. Melanoma can be found in bone, but it would be considered a metastatic lesion and not a primary malignant bone tumor. (Sarwark, Ed., Essentials of Musculoskeletal Care, 4 th Edition, 2010)

What is the first type of motion that is lost as a person develops progressively worsening osteoarthritis of the hip joint? A Abduction B Adduction C External rotation D Flexion E Internal rotation

Explanation The Correct Answer is: E The earliest sign of the development of osteoarthritis of the hip is often the loss of internal rotation. As the condition worsens, muscular contractures may develop which hold the affected limb in a flexed and externally rotated position, which has great consequence on the individual's gait and functioning level. Typically a person experiencing this type of osteoarthritis will eventually develop an antalgic gait where the time spent bearing weight on the affected limb is brief due to pain. Also, the gluteus medius (which is a hip abductor and helps stabilize the pelvis) may become weakened as the condition worsens, resulting in an abductor lurch as the trunk of the body sways out over the affected limb when attempting to walk. (Sarwark, Ed., Essentials of Musculoskeletal Care, 4 th Edition, 2010)

You are evaluating a patient who is having decreased sensation to his arm after he sustained a head on injury while playing football. He is stating that his right arm has decreased sensation that goes into the hand, but he is slightly vague on being descriptive. As you perform the physical examination, what part of the body would exhibit sensation for the C7 component of the brachial plexus? A Lateral forearm B Lateral upper arm C Medial forearm D Medial upper arm E Third Finger

Explanation The Correct Answer is: E The sensory nerve emanating from the C7 level is responsible for sensation of the third finger. The lateral forearm is supplied by C6. The lateral upper arm is supplied by C5. The medial upper arm is supplied T1 and the medial forearm is supplied by C8.

A 45-year-old woman with recent diagnosis of rheumatoid arthritis has begun treatment with celecoxib. She has been on this medication for 3 months and notes that her pain continues. Early signs of joint involvement are present in the patient's hands. Which of the following medications is the most appropriate to add to her treatment? A aspirin B rituximab C etanercept D leflunomide E methotrexate

Explanation The Correct Answer is: E The treatment of rheumatoid arthritis (RA) is aimed at reduction of pain, preservation of function, and prevention of deformity. Although non-steroidal anti-inflammatory drugs (NSAIDs) provide symptomatic relief, they do not alter progression or prevent erosion of the joint. Consequently, in addition to NSAID therapy, disease-modifying anti-rheumatological drugs (DMARDs) should also be initiated as soon as the diagnosis is confirmed. The most common initial DMARD used as treatment of choice in RA is methotrexate. Aspirin should not be added because of the increased risk of gastrointestinal side effects as well as having no effect on altering RA disease progression. Rituximab is a biological DMARD and is indicated to be added in patients with RA refractive to treatment with combination therapy of methotrexate and a tumor necrosis factor inhibitor (TNF). Etanercept is a TNF inhibitor. This class of medication is often added in patients with RA who are not responding to methotrexate therapy alone. Leflunomide is a pyrimidine synthesis inhibitor that is approved for the treatment of RA; however, it is contraindicated for use in premenopausal women secondary to its carcinogenic and teratogenic potential. (Hellmann and Imboden, 2008, pp. 722-725) Hellmann DB , Imboden JB. Arthritis and musculoskeletal disorders. In: Tierney LM , McPhee SJ , Papadakis MA, eds. 2008 Current Medical Diagnosis and Treatment. New York, NY: McGraw-Hill; 2008:703-756.

52-year-old man with hypertension associated with recent unexplained weight loss presents with fever, malaise, and gradual onset of pain and weakness of his leg muscles for the past month. Physical examination reveals a mottled reticular pattern overlying portions of both calves and an area of ulceration with surrounding induration on the left lateral malleolus. Initial laboratory results reveal mild normochromic anemia, leukocytosis, and elevation of C-reactive protein, BUN, and creatinine. Which of the following is the most appropriate diagnostic evaluation to confirm the suspected diagnosis? A HLA-B27 typing B rheumatoid factor C MRI of sacroiliac joints D antinuclear antibodies test E tissue biopsy of area of induration

Explanation The Correct Answer is: E This patient most likely has polyarteritis nodosa (PN). A major obstacle in making the diagnosis is the absence of a disease-specific serological test. The diagnosis requires confirmation with either a tissue biopsy or angiogram. HLA-B27 antigens are not associated with the suspected diagnosis. While classic PN will have low titers of rheumatoid factor and antinuclear antibodies, both are nonspecific findings and will not confirm the diagnosis. An MRI of the sacroiliac joints is indicated in evaluation of the early stages of suspected ankylosing spondylitis and plays no role in the evaluation of PN. (Hellmann and Imboden, 2008, pp. 738-739) Hellmann DB , Imboden JB. Arthritis and musculoskeletal disorders. In: Tierney LM , McPhee SJ , Papadakis MA, eds. 2008 Current Medical Diagnosis and Treatment. New York, NY: McGraw-Hill; 2008:703-756.

A 65-year-old woman presents to the office with decreased hearing, and pain over her sternum, pelvis, and her right tibial tubercle. On x-ray, the involved bones are noted to be expanded and denser than normal. Her serum calcium and phosphorus levels are normal, but serum alkaline phosphatase level is markedly elevated. Which of the following would be the appropriate initial treatment for this patient? A ibuprofen 600 mg po every 6 hours B indomethacin 25 mg po tid C meclizine 25 mg po tid D methotrexate 7.5 mg po qd E tiludronate 400 mg po qd

Explanation The Correct Answer is: E This patient's signs and symptoms are consistent with Paget disease of bone. Biphosphates have become the treatment of choice for this disease. Tiludronate, taken orally for 3 months, is very effective in treatment of this disease.

Which of the following would be the best type of exercise to do to strengthen bones and prevent osteoporosis? A Ballistic stretching B Recumbent bike C Static stretching D Swimming E Weight training

Explanation The Correct Answer is: E Weight bearing and resistance exercises are the most effective at maintaining bone density. Bones respond favorably to regular impact loading exercises such as walking and strength training with either machines or free weights. Adding balance training exercises like those done in tai chi, yoga, or Pilates would be beneficial as well as part of a fracture prevention plan. Ballistic stretching is a type of flexibility exercise that uses bouncing movements to try and increase range of motion. Static stretching is a more gentle approach to stretching where the person stretches as far as they comfortably can and hold that position for several seconds. There are various schools of thought on the benefits and safety of each stretching method. While both likely have a role in maintaining flexibility and function, neither is likely to play a significant role in the prevention of osteoporosis. Exercising on a recumbent bike likely offers some strength and cardiovascular benefits, especially if utilized as part of a strenuous workout, but as a non weight bearing exercise (seeing as the person using the bike is sitting and leaning back with their weight fully supported), this is not the ideal exercise for osteoporosis prevention. Swimming is a wonderful exercise choice since it affords a challenging cardiovascular workout, uses all of the muscles of the body, improves range of motion and is well tolerated by people with various joint ailments, but because it is not a weight bearing exercise, it should be combined with weight training for optimal bone health. (Sarwark, Ed., Essentials of Musculoskeletal Care, 4th Edition, 2010)

A 64-year-old male presents with right knee pain and stiffness while walking and going up and down stairs for the past six months. He states the pain persists all day long and is relieved with rest. There is no history of trauma and his past medical history is unremarkable. An x-ray of the right knee is performed and the following is observed: What is the suspected diagnosis based on the imaging above? A Bursitis B Osteoarthritis C Torn posterior cruciate ligament D Oblique tibia fracture E Spiral femur fracture

Explanation The correct answer is (B). Osteoarthritis (OA) is the most common cause of chronic knee pain over 45 years of age and commonly seen in weight-bearing joints when walking or climbing stairs. As disease progresses, it becomes continuous and present at night. OA in the x-ray shows a narrowed joint space (white arrow) on the medial side of the knee (B); sclerosis of the bone in the medial compartment (black arrow), which is evidence of cortical thickening; and formation of osteophytes in the medial femur (white wedge). Soft tissue injuries including a PCL tear (C) and bursitis (A), would not usually be identifiable on x-ray and would need further imaging, such as a CT or MRI. No fractures, (D) and (E), are identified on this image.

A 6-year-old female child presents with complaints of chronic hip pain so severe that she has not been able to walk to the school bus. Examination shows severe tenderness at the left hip with markedly decreased active and passive range of motion. Radiologic examination demonstrates joint effusion with widening. Which of the following is the most likely diagnosis? A osteochondritis dissecans B slipped capital femoral epiphysis C septic hip arthritis D Legg-Calvé-Perthes disease

The Correct Answer is: D Legg-Calvé-Perthes disease is also known as avascular necrosis of the proximal femur. It typically occurs in children between 4 and 8 years old and persistent hip pain is the main symptom. On examination, the clinician notices a limp and/or limitation of motion of the affected hip. Radiologic examination demonstrates the necrosis with effusion and joint space widening with a negative aspirate. Treatment involves surgical hip replacement. Slipped capital femoral epiphysis (SCFE) is due to the displacement of the proximal femoral epiphysis owing to disruption of the growth plate. The head is normally displaced medially and posteriorly relative to the femoral neck. It typically occurs in adolescence, specifically obese males, and can also be associated with hypothyroidism. SCFE usually occurs after direct trauma to the hip or a fall. Patients complain of vague symptoms at first that progress into pain of the hip or of the knee. On examination, there is decreased internal rotation of the hip that can be confirmed by lateral X-ray of the hip. Septic hip arthritis is not common in children between the age of 5 and 12 years. The legs are held in external rotation to minimize pain and will have a positive aspirate. Osteochondritis dissecans typically presents in the knee, elbow, and talus and is characterized by a wedge-shaped necrosis of bone.

A 67-year-old man presents with pain and stiffness in his shoulders and hips lasting for several weeks with no history of trauma. He also has complaints of headache, throat pain, and jaw claudication. It is imperative to diagnose this patient promptly in order to prevent which of the following complications? A anemia B cerebral aneurysms C mononeuritis multiplex D ischemic optic neuropathy E respiratory tract complications

The Correct Answer is: D The most urgent need for diagnosis of a patient with symptoms of polymyalgia rheumatica (PMR) and giant cell arteritis is to prevent blindness caused by ischemic optic neuropathy as a result of occlusive arteritis of the ophthalmic artery. Early diagnosis is imperative as the neurological damage to the optic nerve is not reversible. Most patients with this diagnosis will have a normochromic-normocytic anemia, but this does not create urgency in treatment. Cerebral aneurysms are not common findings with PMR; large vessels such as the subclavian and aorta may be involved in giant cell arthritis in 15% of patients. Mononeuritis multiplex commonly presents with painful paralysis of a shoulder, and respiratory tract complications are more nonclassic findings with the presentation of PMR.

A parent brings her child into the office with the concern of the way he stands. To the parent the child looks like they are "bowlegged." If the child is indeed bowlegged, what would be the best description of this orthopedic abnormality? A Angulation of an extremity at a joint with the more distal part angled anteriorly. B Angulation of an extremity at a joint with the more distal part angled away from the midline. C Angulation of an extremity at a joint with the more distal part angled toward the midline. D Angulation of an extremity at a joint with the more distal part angled posteriorly. E Angulation of an extremity at a joint with the more proximal part angled away from the midline.

The correct answer is (C). A varus deformity involves angulation of an extremity at a joint with the more distal part angled toward the midline. This is the type of deformity found in someone who is described as "bow-legged."

A 36-year-old auto mechanic presents to the emergency department after hurting his back on the job. While lifting an object, he experienced sudden pain in his lower back with radiation to the right buttock. He was initially treated for muscle strain with a nonsteroidal anti-inflammatory drug (NSAID) after x-rays of his lumbosacral spine demonstrated no pathology. He continued to complain of this low back pain now radiating posteriorly down his left leg to the mid-thigh. Physical examination is unremarkable. The most likely diagnosis is A lumbosacral strain B left S1 radiculopathy C cauda equina syndrome D L5-S1 disc herniation E lateral femoral cutaneous neuropathy

xplanation The Correct Answer is: A Low back pain is one of the more common presenting neurologic complaints to a primary care provider. Most acute pain syndromes are benign, self-limiting conditions, with pain arising from myofascial sources. Patients with back pain and normal neurologic examinations are unlikely to have any serious underlying pathology and further diagnostic testing is usually unrevealing.

A 21 year-old female presents to the emergency department after having a cast applied to her right arm earlier that day. Approximately one hour ago she began having extreme 10/10 pain in her right arm and is in visible distress. When considering the diagnosis of compartment syndrome, permanent damage to the muscle begins after how many hours of ischemia? A >2 hours B >6 hours C >8 hours D >12 hours E >24 hours

xplanation The Correct Answer is: C Permanent damage results after >8 hours (C) of ischemia. Nerves begin to lose conduction within 2 hours of onset of elevated pressures. Neurapraxia can occur within 4 hours, and irreversible damage occurs 8 hours after elevated pressures. Functional impairment is unlikely when compartment syndrome is diagnosed and treated within 6 hours of its onset. While deficits can occur prior to 8 hours, permanent damage is usually not seen before 8 hours (A and B). After 12 or 24 hours, permanent damage has already resulted in most cases (D and E). The key to early detection of compartment syndrome is a high index of clinical suspicion.


Set pelajaran terkait

Real Estate, Chapter 2, Section 3 (Leasehold Estates), Pop Quiz Questions 3.1

View Set

Multinational Finance: Financial Goals and Corporate Governance

View Set

Chapter 9: Embracing Diversity and Inclusions

View Set

Latitude & Longitude (fill in the blank)

View Set

Chapter 10 finance test materials

View Set

Chapter 3: Ethics, Law, and Delegation in Nursing

View Set